NCCT- Patient Care

अब Quizwiz के साथ अपने होमवर्क और परीक्षाओं को एस करें!

The physician has ordered a nasal inhaler for a patient with seasonal allergies. The medical assistant is instructing the patient on proper use. Which of the following statements from the patient indicates proper understanding of the technique?

"I will shake the inhaler prior to use." Rationale When a physician orders a medication to be administered via an inhaler, the medication is intended to be delivered to the lungs. Since the inhaler in this case is a nasal inhaler (as opposed to a mouth inhaler like many of those used to treat asthma), the nose should be clear of obstruction and should not be blown immediately after administering the medication to refrain from inhibiting medication from reaching the lungs. Patients should shake an inhaler to thoroughly mix the contents, hold it upright, and breathe in the medication.

A patient calls the physician's office talking loudly, demanding to know why he was billed personally for his check-up last month. Which of the following statements is an appropriate response by the medical assistant in handling the angry caller?

"Let me pull up your record and see how I can help." Rationale Managing angry clients can be challenging. A professional and proactive approach is to stay calm and use the LAST model (Listen, Apologize, Solve and Thank). Listen to the caller's concern, apologize for the situation, solve the issue if possible (this may involve pulling the records to gather details), and thank the caller for working through this. Don't immediately sidestep the issue and send the caller to contact the insurance company (even though that may eventually be the only thing to do; try to assist the patient first). Don't dismiss the concern by telling them not to worry and you're sure insurance will cover it. Address the patient's concern during the call, it is not good practice to delay action by telling the caller to bring the bill to the next appointment.

The patient asks the medical assistant why vital signs are taken at every office visit. Which of the following is the best response?

"To obtain a baseline measurement of your overall health status." Rationale This response provides a direct answer to the patient's question. Letting them know that a baseline is necessary, also communicates that the Physician uses it to provide proper care of the patient. The other responses, while possibly true, would not be an adequate response to their question.

The medical assistant is interviewing an apprehensive patient complaining of persistent lower back pain. Which question puts the patient at ease and encourages open communication?

"What has been going on with you since your last appointment?" Rationale The medical assistant should ask the apprehensive patient what has been going on since their last appointment. This approach requests the patient to describe their complaints beyond a yes/no answer in an open, non-defensive manner. This question will put the patient at ease and allow them to openly communicate about their problem and anything else that may be bothering them. Asking what medication they took to help with the pain and asking "why" questions don't encourage open communication.

When explaining a procedure to a pediatric patient, which of the following statements by the medical assistant puts the preschool child most at ease?

"You are going to go into a special room, and we are going to take a picture of your arm." Rationale The medical assistant should use a minimally threatening language and positive tone in order to reduce fears of the child. By telling the child that the arm is going to be squeezed, they're getting poked with a needle, or the medicine is going to sting will upset the child.

Which of the following suffixes means removal?

-ectomy Rationale The suffix -ectomy means removal of. Ex. Appendectomy - surgical removal of appendix. The suffix -otomy means to cut, -itis means inflammation of, and -scopy is when the healthcare provider looks at and observes something in the body.

The medical assistant is performing a visual acuity test in the clinic using a Snellen eye chart. The patient should be positioned away from the eye chart at a distance of

20 feet. Rationale Average distance visual acuity is 20/20 vision, meaning the eye should see an object at 20ft. away. Ex. Having 20/40 vision means the patient must be at 20 feet to see what a person with normal vision can see at 40 feet.

Which of the following indicates the normal resting heart rate for an adult?

60-100 bpm Rationale In addition to age, factors such as activity level, gender, size, physical condition, presence of disease, medications, or mood will influence pulse rate. Clinical Pearl: As age increases, heart rate decreases.

Which of the following is considered the normal pulse range for an adult patient?

60‑100 bpm Rationale A normal pulse range for an adult is 60-100 bpm. A HR<60 is referred to as bradycardia, and a HR>100 is known as tachycardia.

A patient's weight of 155 lbs. is equivalent to

71 kg. Rationale [Formula conversion: pounds to kg 1 kg = 2.2 lb] 155 lbs./2.2 kg=71kg

An adult patient has a second degree burn that covers his entire left arm. Which of the following best describes the estimated surface area involved based on this information?

9% Rationale All burns are measured by two distinct measurements: the affected body surface (percentage) and the thickness and depth (degree) of the burn. Since this instance involves a second degree burn, it is necessary to find the affected body surface. A quick and easy way to do this is by applying the Rule of Nines (accurate for adults, but not children): all body parts are divisive by 9. An entire arm front and back is 9%. The entire head is 9%, the front torso is 18%, back torso is 18%, and each leg is 18%. The remaining 1% refers to the perineum.

Which of the following findings is most likely to be present at the site of a tick bite in a patient with suspected Lyme disease?

An apparent bullseye rash, with a large area of redness and clear center. Rationale A typical skin reaction that occurs when bacteria is passed from an affected tick to humans is an apparent bull's-eye rash (centered around the bite), with a large area of redness and clear center. Signs and symptoms of Lyme disease include fever, chills, fatigue, and joint pain. There are many types of rashes and they can be difficult to tie to one particular disease or reason for manifestation. A butterfly rash is generally seen in people with the autoimmune condition Systemic Lupus Erythematosis (SLE). Silvery scales are often associated with Psoriasis. Small clear vesicles can be a sign of chicken pox.

The patient experiencing a nose bleed presents to the reception area in the medical office. Which of the following actions should the medical assistant take?

Apply direct pressure to the affected nostril for 10-15 minutes. Rationale Applying direct pressure to the affected nostril for 10-15 minutes will generally stop the bleeding and allow clotting to take place. None of the other options listed in this scenario allow the blood to clot, therefore the patient would keep losing blood.

The closure of which valve(s) within the heart create(s) the "lub" sound of the heart beat's "lub dub" cycle?

Atrioventricular valves Rationale With a stethoscope, the first heart sound (S1=lub) can be heard the loudest at the mitral area, with the closure of the mitral (bicuspid) and tricuspid valves, also known as the atrioventricular valves. It is a dull, low pitched sound marking the beginning of ventricular systole. The second heart sound (S2=dub) signifies the beginning of diastole, caused by the semi lunar valves closing. S3 and S4 are abnormal heart sounds called "gallops". S3 occurs in early diastole, with the first phase of rapid ventricular filling. S4 occurs in late diastole, with atrial contraction.

What is the medical term associated with a decrease in size or wasting away of muscle tissue?

Atrophy Rationale The decrease in size of a muscle is called atrophy. Edema refers to the increase of fluid of a body part and is characterized by swelling. Sclerosis occurs when the tissues get hard and stiff, and necrosis is when cells and tissues die.

While obtaining vitals on an infant, how is the heart rate measured?

Auscultate the apical pulse for 60 seconds. Rationale Apical pulse is used for an infant because the radial pulse is difficult to feel (plus an infant's pulse is generally irregular). The apical pulse is ausculted (the health professional will listen with a stethoscope) for one full minute. Brachial, radial and femoral pulses are difficult to feel on an infant.

The medical assistant is preparing a patient to have cryosurgery for lesions on the cervix. Which of the following should the medical assistant include in post-op instructions?

Be prepared for mild cramping and bleeding. Rationale Cryosurgery is done by freezing off the lesions of the cervix. Therefore, the medical assistant should tell the patient to expect mild cramping and bleeding while the dead tissue is leaving the body. The procedure is typically done in a clinic and should only take about 15 minutes. The patient should not have to be on bed rest after the procedure and should be able to return to work right away. Normal eating habits can also resume after the procedure.

Referring to the Illustration, identify number 91. (inside upper arm muscle)

Biceps brachii Rationale Basic anatomy, rationale not applicable.

The physician orders a nebulized breathing treatment for a patient with asthma. The medical assistant will use which of the following types of medication when administering this treatment?

Bronchodilator Rationale A bronchodilator is used to decrease the work of breathing by relaxing smooth muscles and opening large and small airways. A mucolytic is designed to help loosen and clear the mucus from the airways by breaking up the sputum. An antihypotensive agent, also known as a vasopressor agent, is any medication that tends to raise reduced blood pressure. An expectorant is a medication that helps bring up mucus and other material from the lungs, bronchi, and trachea.

Which of the following diagnostic abbreviations means "stroke"?

CVA Rationale The abbreviation for stroke is CVA, which stands for cerebrovascular accident. The abbreviation MI stands for myocardial infarction, which is also known as a heart attack, CHF stands for congestive heart failure, and CAD stands for coronary artery disease.

Which of the following is the result of a ruptured or occluded blood vessel?

CVA Rationale ​A cerebrovascular accident (CVA) is the medical term for a stroke. A stroke occurs when blood flow to a part of the brain is occluded due to a blockage or rupture of a blood vessel. COPD (chronic obstructed pulmonary disease) is a progressive disease in which the patient has difficulty breathing. CF (cystic fibrosis) is a genetic disorder affecting the lungs. A patient with GERD (gastroesophageal reflux disease) suffers from stomach acids traveling back up into the esophagus irritating the its lining.

Referring to the Illustration, identify number 76. (collarbone)

Clavicle Rationale Basic anatomy, rationale not applicable.

Which of the following describes the correct technique for cleansing a patient's minor wound?

Cleanse from the inside of the wound toward the outside. Rationale To clean an open wound, wipe using circular motions from the center of the wound outward. Be sure to clean at least 1" beyond the wound margins. This is to prevent infection and promote healing. Never clean from the outside towards the inside. Depris and contaminants can be carried into the wound and lead to infection. Once the wound itself has been dealt with, cleansing in the areas around the wound would then be appropriate to check for possible further injury.

Which of the following are outside of an entry level medical assistant's scope of practice?

Collect ABG specimens. Rationale Medical Assistants are allowed to remove surgical staples, perform peak flow testing, and test specimens for occult blood, but it is not in their entry level scope of practice to collect ABG specimens (this is an arterial blood draw). ABG specimens are typically collected by specially trained personnel.

The medical assistant discovers that one end of an unopened, disposable sterile laceration tray was placed in a small area of water by the sink. Which of the following actions should the medical assistant take?

Discard the package as it has been contaminated. Rationale The medical assistant should discard the package, because moisture provides a medium for bacteria growth, therefore the package is no longer considered sterile. Even though the tray wasn't opened, it's still considered contaminated since it was in the water. All contaminated objects need to be discarded to prevent any infections from occurring.

Which of the following should a patient do when using a Holter monitor? (Select the three (3) correct answers.)

Eat meals on a regular schedule., Resume a normal daily routine., Keep a log of daily activities. Rationale When a patient has on a Holter monitor, he/she should continue on a regular schedule. The patient should keep the daily activity log book given to him/her so the medical professional can match up an activity to heart activity. Medication should not be stopped during this time.

Which of the following interventions by the medical assistant is appropriate in preparing the patient for cervical cryosurgery?

Encourage the patient to take slow, deep breaths to promote muscle relaxation. Rationale Cryosurgery is a procedure that is used to kill cancerous cells in the cervix by freezing them off. It is a quick procedure done in the doctor's office. The medical assistant should encourage the patient to take slow, deep breaths to minimize stress and promote relaxation of the patient's pelvic muscles. Cramping is common during the procedure, so it is important that the patient take slow, deep breaths. Monitoring the heart rate isn't necessary during the procedure, and the patient should be in the lithotomy position with the feet up in stirrups. A sterile skin prep in the lumbar region isn't necessary, because the procedure is done entirely in the cervix.

The patient is diagnosed with hepatomegaly. Which of the following describes this condition?

Enlarged liver Rationale Hepatomegaly is swelling of the liver beyond its normal size. The size and weight of your liver increases with your age and body weight. Sex and body shape also influence the size of your liver. Hepatomegaly is always a cause for medical evaluation. An Engorged spleen or Gall stones would not apply in this diagnosis. Reflux disease is also known as gastroesophageal reflux disease (GERD) and affects the lower esophageal sphincter.

A pediatric patient with no known seizure history presents to the physician's office with a high fever. While sitting in the waiting room, the child goes unconscious and begins to experience tonic/clonic movements that do not stop after several minutes. Which of the following actions should the medical assistant take?

Ensure immediate patient and environment safety and activate EMS. Rationale The medical assistant should make sure that the area is safe and then activate EMS. Emergency care is indicated, because the patient's condition is or could potentially become life threatening and/or cause injury. Putting a cool wash cloth on the child is not the priority intervention, because it doesn't protect the child from the seizure they're experiencing, and placing a tongue blade in the mouth could cause injury. Obtaining a full set of vitals wouldn't be appropriate, because the patient is having a seizure and wouldn't be able to cooperate with the medical assistant.

Which of the following actions by the medical assistant is appropriate when interacting with a patient who is visually impaired?

Explain where things are positioned by referencing a clock image. Rationale By providing general orientation to the environment, the visually impaired patient feels more comfortable. It is of utmost importance to maintain a visually impared patients dignity and independence in this type of setting. Guiding them physically takes away independence and could decrease their comfort level. Limiting communication or talking loudly is inappropriate.

Referring to the Illustration, identify number 77.(Upper leg bone)

Femur Rationale Basic anatomy, rationale not applicable.

Referring to the Illustration, identify number 74 (front of leg bone)

Fibula Rationale Basic anatomy, rationale not applicable.

In which of the following positions should the medical assistant position the patient who is experiencing dyspnea?

Fowler's Rationale Dyspnea indicates that the patient is having difficulty breathing. If this occurs, the patient should be placed in an upright (Fowler's) position (at 60-90 degrees) allowing for maximum lung expansion. Both the supine position (a position in which the body is lying with the face up) and the prone position (in which the body is lying face down, sometimes with the hands behind the head or neck) could cause more breathing constriction than Fowler's. The dorsal recumbent position is used for examination of the abdomen, occasionally for vaginal or rectal exams, and for some surgical procedures.

An older adult patient with COPD and on continuous oxygen per nasal cannula presents for a physical exam. Which of the following positions should the medical assistant place the patient in?

Fowler's Rationale The patient should be in an upright position (Fowler's) to allow for maximum lung expansion and optimal oxygenation. Sims position is when the patient lies on one side and flexes the opposite knee, dorsal recumbent is when the patient lies on their back with the knees bent, and knee-chest position is when the patient's weight is on the knees and the hips are in the air. None of these three options facilitate optimal oxygenation for the patient with COPD.

The abbreviation for gastrointestinal is

GI. Rationale GI is the abbreviation for gastrointestinal, GU is the abbreviation for genitourinary, gyn refers to gynecology, and abd is the abbreviation for abdomen.

Which of the following foods should the medical assistant recommend to a patient inquiring about ways to increase their dietary intake of folic acid?

Green leafy vegetables, fruits, organ meats, and dried yeast. Rationale Folic acid is an essential B vitamin contributing to neurologic health and brain development. Foods that are high in folic acid are green leafy vegetables, fruits, organ meats (from the organs- liver, heart, etc.), dried yeast, and beans. Dairy and eggs contain amino acids, vegetable seed and oils contain essential fatty acids (like omega 3 and omega 6) and whole grains & legumes contain abundant in essential fatty acids, vitamin E, B-group vitamins, minerals and phytonutrients.

Referring to the Illustration, identify number 71 (back of upper arm)

Humerus Rationale Basic anatomy, rationale not applicable.

Which communication approach describes the ability to see things from another person's point of view?

Identifying with or being sensitive to another person's feelings or problems is an essential component of therapeutic communication. Sympathy or "feeling sorry" for the patient requires emotional involvement that oversteps the professional boundary, and does not encourage expression of feelings.

Which of the following charts should the medical assistant use to assess color blindness?

Ishihara Rationale Color vision acuity is screened by administering the Ishihara test (booklet or cards). Snellen chart screens visual distance acuity. Jaeger system screens near vision acuity. Random Dot E test screens for visual depth and perception in small children (helps identify problems like strabismus and amblyopia).

Identifying 10 out of 14 color pages is considered within normal limits for which of the following visual acuity exams?

Ishihara Rationale Color vision screening should be performed in a well-lit room, using the Ishihara screening book or cards. Color vision deficits are more prevalent in males. The Jaeger test measures how near or far you can read text based on the size of the typewritten words. The Snellen test uses letters to determine the size of type that a patient can read at a certain distance (this is the standard chart with the "E" at the top). A Random Dot E test generally determines depth and distance perception (a.k.a. stereo acuity).

The medical assistant is preparing a 68-year-old male for a lumbar puncture. Which of the following explains why the patient must keep the knees drawn up to the abdomen?

It reduces trauma to the spinal cord and widens the spaces between the lumbar vertebrae. Rationale When a lumbar puncture is performed, the technician must puncture the dura, the membrane that contains the brain and spinal cord, as well as the cerebrospinal fluid that they are suspended in. If a patient is lying flat but knees are not bent, the subarachnoid space is not opened. Up to 25 percent of people who have undergone a lumbar puncture develop a headache afterward due to a leak of fluid into nearby tissues. The headache typically starts several hours up to two days after the procedure and may be accompanied by nausea, vomiting and dizziness. The headaches are usually present when sitting or standing and resolve after lying down

In which position does the medical assistant assist the patient into just prior to a pelvic exam?

Lithotomy Rationale The Lithotomy position facilitates the provider's examination of pelvis and specimen collection. It allows for ease of vaginal speculum insertion and maximum visualization during a pelvic exam. The Trendelenburg position may be used to prevent and treat shock, for radiologic exams and procedures, and for some types of surgery. Trendelenburg position has the patient lying supine, but with the feet/pelvis higher than the head (as much as 45 degrees). The Sims' position (when the patient lies on one side and flexes the opposite knee), is also known as the left lateral position. Sims' may be used for administering rectal suppositories and enemas, certain medical exams and surgical procedures. The supine position is a position in which a patient is lying on the back with the face up.

Which of the following abbreviations means "heart attack"?

MI Rationale MI is the abbreviation for myocardial infarction. The myocardial is the heart muscle and infarction is the lack of oxygen leading to tissue damage/death. Therefore, a myocardial infarction (MI) is a heart attack. HTN is the abbreviation for hypertension, which is high blood pressure. CABG is the abbreviation for coronary artery bypass graft, which is a procedure used to treat coronary artery disease. The abbreviation DVT stands for deep vein thrombosis, which is when a blood clot forms in a vein.

A patient diagnosed with allergies and a UTI has an elevated leukocyte count reflected on the CBC laboratory report. Which particular leukocyte populations would be elevated?

Neutrophils/Eosinophils Rationale Leukocytes are white blood cells. There are many types of white blood cells: lymphocytes, monocytes, neutrophils, eosinophils, and basophils. Neutrophils are often elevated in the presence of bacterial infection. Eosinophils are often elevated in the presence of allergic reactions. The patient in this case has allergies and a urinary tract infection (UTI), so neutrophils and eosinophils would both be elevated.

A patient presents with drainage from both eyes. The medical assistant understands that which of the following is the correct abbreviation for "both eyes"?

OU Rationale OU=both eyes OS=left eye OD=right eye BO is a abbreviation having several indications such as: basioccipital, bowel obstruction, body odor, etc. thus the words are best spelled out to avoid confusion.

Which of the following statements about pain management is true?

Pain is what the patient says it is. Rationale Pain is an individual experience. Neonates (newborns 0-28 days of age) are more sensitive to pain than older children. Untreated pain is a larger problem than the risk of addiction. According to the Joint Commission pain standards, every patient has the right to appropriate pain assessment and management. Always use an objective scale in pain assessment, reassessment, and documentation in addition to the patient's subjective statements describing their pain.

Referring to the Illustration, identify number 78.(Kneecap)

Patella Rationale Basic anatomy, rationale not applicable.

Referring to the Illustration, identify number 92 (breast muscle)

Pectoralis major Rationale Basic anatomy, rationale not applicable

After checking an unconscious patient's vitals, the adult patient has a heart rate of 75 BPM and no signs of breathing. Which of the following is the correct course of action?

Perform rescue breathing. Rationale Since this patient has a pulse but is not breathing, rescue breathing would be the appropriate course of action. If CPR is indicated for an adult patient, there would be no pulse (no measurable heart rate) or signs of breathing. The presence of a heart beat eliminates the need for chest compressions or a defibrillator. Abdominal thrusts would be performed on a patient that is choking, which is not a factor in this case.

Referring to the Illustration, identify number 93 center (inner thigh muscle)

Quadriceps femoris Rationale Basic anatomy, rationale not applicable.

The liver is located in which quadrant of the abdomen?

Right upper Rationale The liver is the largest glandular organ in the body and performs many vital functions to keep the body pure of toxins and harmful substances. An average adult liver weighs about three pounds. Located in the upper-right portion of the abdominal cavity under the diaphragm, the liver consists of four lobes. It receives about 1.5 quarts of blood every minute via the hepatic artery and portal vein.

Which of the following treatments are appropriate when initiating first aid to the patient with a second-degree burn? (Select the two (2) correct answers.)

Run cool water over affected area of intact skin., Protect the burn from pressure or injury. Rationale Cool water should be ran over the affected area of intact skin, and the burn should be protected from pressure injury. Oil or ointment application increases the risk of breaking blisters and increases the potential for infection. Intact skin is a first line of defense in preventing infection, therefore the blisters should not be broken or drained, and Epsom salt would not be used. Previous Question

Referring to the Illustration, identify number 72. (tailbone area)

Sacrum Rationale Basic anatomy, rationale not applicable

Referring to the Illustration, identify number 70. (back of shoulder)

Scapula Rationale Basic anatomy, rationale not applicable.

Which of the following positions should the medical assistant place the patient in when preparing to instill an enema?

Sims Rationale Left Sims position (side-lying, right knee flexed) allows for ease of enema instillation into the sigmoid colon. The supine position is a position in which a patient is lying on the back with the face up. Trendelenburg position may be used to prevent and treat shock, for radiologic exams and procedures, and for some types of surgery. Trendelenburg position has the patient lying supine, but with the feet/pelvis higher than the head (as much as 45 degrees). Dorsal recumbent is when the patient lies on the back with knees bent.

The physician has ordered a suppository to be administered to a patient with chronic constipation. The medical assistant should place the patient in which of the following positions?

Sims Rationale When a patient is in the Sims position, he or she lies on one side with one knee bent, making the rectal area accessible for insertion of a suppository. The Trendelenberg positions the patient head down while elevating the feet (this position is widely used to manage hypotensive patients, but the evidence to support its effectiveness is suspect). Supine positioning refers to lying on one's back, face upward, whereas dorsal recumbent refers to lying on the back with knees bent and feet flat.

Which of the following eye tests checks for distance visual acuity?

Snellen Rationale The Snellen test checks for distance visual acuity. The test is performed by having the patient read letters on a chart from a distance and determining the smallest row of letters the patient can read. The Ishihara test checks for color blindness and the Jaeger test evaluates how well the patient sees close up (near vision). The random dot E test checks stereopsis, also referred to as depth perception. The test is performed by having the patient put on polarized glasses. The provider holds a card with an "E" on it. The patient is then asked to say what the object is on the card and from this the provider can determine if the patient has any problems with depth perception.

When performing visual acuity testing, which screening tool is necessary in checking distance acuity?

Snellen chart Rationale Designed for vision testing, the Snellen test checks for distance visual acuity. Represented as a fraction, 20/20 is considered normal visual acuity. The test would be performed at a distance of 20 feet away from chart. To evaluate near vision, the eye doctor may use a small hand-held card called a Jaeger eye chart. The Ishihara test is used to determine color blindness by means of a series of cards each having colored dots that form one pattern to the normal eye and a different pattern to the eye that is color-blind.

While performing a Snellen visual acuity test, the patient misses two letters on the 20/70 line with his right eye. How should the medical assistant document this finding?

Snellen chart OD 20/70-2 Rationale When documenting visual acuity with the Snellen chart, always start with 20. This represents 20 feet as the distance of the patient to the chart. The second number is the line that is read. A "-" represents letters missed.

Which of the following congenital defect involves malformation and incomplete closure of the spine?

Spina Bifida Rationale The malformation and incomplete closure of the spine is known as Spina Bifida, and most cases are detectable on ultrasound by 18 weeks of pregnancy. Klinfelter's Syndrome is when the male is born with an extra X chromosome, Phenylketonuria is when a baby cannot break down a substance called phenylalanine, and if it builds up in the body, it could cause serious health issues. Talipes is another name for clubfoot, which is when the baby is born with a malformation of the foot.

Referring to the Illustration, identify number 75. (center of chest bone)

Sternum Rationale Basic anatomy, rationale not applicable

Which of the following instructions should the medical assistant give a patient prior to a scheduled spirometry PFT test?

Stop using bronchodilators for 24 hours. Rationale Medical assistants must instruct patient's preparations for the tests prior to the test being done. Spirometry (meaning the measuring of breath) is the most common of the pulmonary function tests (PFTs), measuring lung function, specifically the amount (volume) and/or speed (flow) of air that can be inhaled and exhaled. Since a PFT is a pulmonary function test to check breathing functions, we instruct the Patient to stop smoking, refrain from using bronchodilators or nebulizers for 24 hours prior to test (depending upon the drug in use, this could be 6 hours).

Referring to the Illustration, identify number 69. (side of head)

Temporal Rationale Basic anatomy, rationale not applicable.

A pediatric patient's radiograph shows a greenstick fracture. This indicates which of the following?

The bone is bent and only partially broken. Rationale There are many different classifications of fractures. A greenstick fracture is often caused by compression forces. This type of fracture causes the bones (usually the long bones in children) to crack on one side, remaining intact on the other side. The bone is flexible and could slightly bend, thus only resulting in a partial break. Splintered (comminuted) fractures happen when the bone is broken into several small pieces. An oblique fracture is a fracture that slants through the bone. An open or compound fracture indicates that the bone is broken through the skin and is open to the external environment.

When instructing a patient on the proper way to perform a self-breast exam, which of the following statements is correct?

The exam can be performed lying down or standing up. Rationale The medical assistant should instruct the patient that the exam can be performed lying down or standing up, and they should use the finger pads to feel the breast for any abnormalities. The exam should be performed consistently at the same time each month in order to identify any changes, and any unusual findings should be reported to a physician immediately.

In the image, click on the best anatomical site for auscultating an apical heart rate:

The medical assistant would listen with the stethoscope at the apex (mitral valve) of the heart, located in the left, fifth intercostal space, midclavicular line. This is where the pulse can best be heard. The aortic valve is used when listening for a disease of the valve, and aortic valve stenosis, which is when the aortic valve starts to narrow. The medical assistant should listen to the pulmonic valve if pulmonary valve stenosis is suspected. Pulmonary valve stenosis is indicated when the pulmonary valve narrows and can be detected by a murmur. Tricuspid stenosis, or tricuspid regurgitation, can also be detected when listening to the tricuspid valve. These will also cause a murmur or an extra heart sound.

The medical assistant auscultates an apical pulse of 70 bpm in an older adult patient who is taking digoxin (Lanoxin). How should the medical assistant interpret this finding?

The patient's heart rate is within normal limits. Rationale Digoxin works to slow and control the heart rate, and a normal heart rate is between 60 and 100. This patient's heart rate is within normal limits, but Digoxin should not be given if a patient's baseline apical heart rate is less than 60 bpm. Tachycardia would be a heart rate greater than 100 bpm, bradycardia would be a heart rate less than 60 bpm, and a pulse deficit is when the pulse cannot be found because the heart isn't contracting properly.

Which of the following instructions should the medical assistant include in a teaching plan for correct use of crutches? (Select the four (4) correct answers.)

Top of crutches should be 1-2 inches below the axilla when standing., Use hands to support body weight rather than the axilla., Hand grips should be even with the hip., Take slow, small steps. Rationale Patients have varying skill levels when it comes to using crutches. It is important to teach patients how to properly adjust and use these mobility aids. The top of the crutches should be 1-2 inches below the axilla when patients are in the standing position. Patients should use their hands to support their body weight instead of the leaning on the axilla. The hand grips on the crutches should be even with the hips. Patients should take slow, small steps. The top of the crutches shouldn't fit snugly in the axillary space because this would irritate that area.

Referring to the Illustration, identify number 73. (back of lower leg)

Tibia Rationale Basic anatomy, rationale not applicable.

The abbreviation meaning treatment is

Tx. Rationale The abbreviation for treatment is Tx. Fx is the abbreviation for fracture, Rx is the abbreviation for prescription, and Sx can either be the abbreviation for symptoms or surgery.

The medical term meaning movement of a body part away from the body is

abduction. Rationale The prefix ab- means away from, therefore abduction would mean to move away from the body. Adduction means to move the body part towards the body, dorsiflexion means to flex the limb backward or upward, and pronation means to rotate the hand so the palm is facing downward, and the weight would be placed on the inside of the foot.

The expected time frame to obtain a reading when measuring a temperature with a digital thermometer is which of the following?

a few seconds Rationale A digital thermometer will signal quickly when a stable temperature reading is registered. Generally this happens within a few seconds (and less than 30 seconds). The temperature may disappear if the clinical professional does not read it within a timely fashion.

Auscultation is a method of examination that requires the use of

a stethoscope. Rationale A stethoscope is a medical instrument that is used to listen to sounds within the organs (auscultation), whereas an otoscope is a medical device used to peer into the ears. A sphygmomanometer measures blood pressure, and a percussion hammer is utilized to test reflexes.

After receiving the MMR vaccine, a 12-month old begins coughing, wheezing, and breathing appears labored. The patient is most likely experiencing

a systemic allergic reaction to the vaccine. Rationale These symptoms are likely a systemic allergic reaction to the MMR vaccine. A shot check should always be performed and documented 20 min following an immunization injection to assess for s/sx of an allergic reaction. A localized allergic reaction to the vaccine would cause a reaction where the vaccine was inserted, but a systemic reaction would cause a reaction all over the body. An apneic episode is when breathing stops, and seasonal influenza would have other symptoms, such as a fever or body aches.

The medical term myoma indicates the patient has

a tumor in the muscle. Rationale A myoma is a non-malignant tumor in the muscle. Myo= muscle; oma= tumor; Mammary= breast; megaly= enlargement

A hernia may be described as

a weakness in the muscle wall allowing underlying tissue to push through it. Rationale A hernia is an abnormal protrusion that occurs when the muscle wall is weakened, allowing underlying tissue to push through. Ex. Hiatal hernia, Inguinal hernia, Umbilical hernia. Sepsis is when an infection has spread to the blood, diverticulitis is when pouches form on the bowel wall and become infected, and Myasthenia Gravis is an autoimmune neuromuscular disease that causes musculoskeletal weakness.

The abbreviation ADLs means

activities of daily living. Rationale ADL is the abbreviation for the activities of daily living. These activities include tasks such as bathing, dressing, and grooming. Ex. A patient's ability to perform ADL's is assessed prior to discharge from an acute care facility. Previous Question Next Question

The patient presents to the clinic during walk-in hours with an apparent first-degree sunburn. Which of the following types of pain is commonly associated with this condition?

acute Rationale Acute pain begins suddenly, can be severe, and lasts a relatively short time (up to 4 weeks). Chronic pain is persistent and ongoing, lasting 6 months or longer (ex. end stage cancer pain, migraine headaches). Phantom pain is associated with the sensation of a missing body part following removal (ex. leg amputation). Referred pain is felt indirectly, in a different location from an injured or diseased body part (ex. back pain with a urinary tract infection).

At which of the following times should the medical assistant instruct the patient to perform a post-prandial blood glucose test?

after a meal Rationale The term post means after, and prandial means meal, therefore a post-prandial blood glucose test should be measured two hours following a meal. A pre-prandial glucose test should be measured before a meal.

Which of the following information should the medical assistant expect to present to the physician from a pulmonary function test?

air flow and volume rates Rationale Pulmonary function tests are a group of tests that measure how well the lungs take in and release air and how well they move gases such as oxygen from the atmosphere into the body's circulation. It will record the lung capacity (volume) and function of air flow. MAs perform spirometry tests and pulse OX; this will diagnosis the extent of lung disease. If a patient has an acid-base imbalance, the doctor may order blood gas tests, which measure the pH and oxygen and carbon dioxide levels in an arterial blood sample, to help evaluate the severity of the imbalance and monitor its response to treatment. Diffusing capacity or transfer factor of the lung for carbon monoxide (CO), is the extent to which oxygen passes from the air sacs of the lungs into the blood. Commonly, it refers to the test used to determine this parameter. Pulse oximetry is a method used to measure the concentration of oxygen in the blood.

A patient comes to the medical office reporting a newly twisted ankle. The appropriate course of action for the medical assistant is to prepare

an ice pack. Rationale In general, a new injury will cause inflammation. A cold application (ice pack, gel pack, cold compress, etc.) will decrease the blood flow to the injury, thereby decreasing the tendency for swelling (after 72 hours, the inflammatory response will likely cease such that no further swelling will likely occur). A cold pack is also used immediately on a new sprain to help limit pain. The general rule of thumb to follow after a soft tissue injury is RICE: Rest, Ice, Compression, Elevation. (Some sources call for Protection, making the acronym PRICE). After 72 hours, heat may be applied. The heat may soothe the area by helping increase blood flow. Heat can be applied locally with a heating pad or warm moist compress. A paraffin wax bath is another heat application that keeps the heat in a localized area utilizing the molding and hardening properties of wax.

Which of the following terms means prolonged lack of appetite?

anorexia Rationale The term anorexia refers to the prolonged lack of appetite. Anemia is when the body doesn't have enough red blood cells, alopecia is hair loss, and insomnia is a term referring to trouble falling asleep or staying asleep.

A patient complains of nausea and vomiting, and has been unable to keep fluids down over the course of two days. Which of the following drug classifications does the medical assistant anticipate will be ordered for this patient?

antiemetic Rationale An antiemetic is used to treat nausea and vomiting. (Ex. ondansetron (Zofran). An antitussive would be a medicinal drug used in those with coughing and related conditions, antibiotics are a type of antimicrobial used specifically against bacteria to treat various infections and antipyretics are substances that reduce fever.

Which of the following organs of the gastrointestinal tract is located in the right lower quadrant of the abdomen?

appendix Rationale RLQ of the abdomen contains the appendix, cecum, right ovary and fallopian tube (females), right ureter, and right spermatic cord (males). RLQ pain (that may or may not be radiating) is most often associated with appendicitis.

During a mole excision, the medical assistant is asked to assist the physician. Prior to handing the physician medical supplies from the sterile field per his request, the medical assistant should

apply sterile gloves. Rationale When assisting the physician, the medical assistant should apply sterile gloves in order to maintain the sterile field. If the medical assistant only applied clean gloves, the sterile field would not be maintained, and the supplies would become contaminated. A second sterile field and getting further assistance may be necessary at some point in the procedure, but the priority action the medical assistant should take is applying sterile gloves in order to keep the patient safe.

Blood pressure measures cardiovascular function by measuring the force of blood exerted on

arteries Rationale Blood pressure is a measurement of the force of blood exerted on the arteries while the heart is pumping blood through the body. The arteries branch out into arterioles, which then branch off into tiny capillaries. The capillaries help facilitate the exchange of water, oxygen, and other nutrients through the body. The capillaries then widen and become venules, which then get bigger to become veins, and veins pump the blood back to the heart to start the process all over again.

Which of the following vessels circulates blood away from the heart?

arteries Rationale Sites used for palpating an artery for pulse rate, quality, and strength include: radial, brachial, carotid, temporal, femoral, popliteal, dorsalis pedis, posterior tibialis. Arteries carry blood away from the heart; veins carry blood toward the heart.

A patient's activity order following surgery is: up ad lib. The medical assistant should inform the patient they may ambulate

as tolerated. Rationale The abbreviation ad lib means to do as desired or tolerated. Four times per day is abbreviated as qid, three times per day is abbreviated as tid, and assistance x2 means the patient needs more than one person helping them do certain tasks.

A pediatric patient with a recent history of chicken pox presents to the clinic with Reye's Syndrome. When reviewing the patient's chart, which of the following is the most likely contributor to the Reye's Syndrome?

aspirin Rationale The cause of Reye's syndrome is unknown, but has been linked to aspirin and generally presents in children who are recovering from a viral infection. The Varicella vaccination would not have caused this syndrome. It is a vaccination for chicken pox and the child in this scenario is recovering from chicken pox. Calamine lotion could have been used to relieve the chicken pox and has not been linked to this syndrome. Acetaminophen is an active ingredient in Tylenol and other NSAIDs (nonsteroidal anti-inflammatory drugs) and has not been linked to Reye's Syndrome.

Which of the following respiratory disorders is manifested by inflammation, swelling, and constriction of the bronchioles and bronchi?

asthma Rationale Asthma is a respiratory disorder that is manifested by inflammation, swelling, and constriction of the bronchioles and bronchi. With an asthma exacerbation, bronchospasms cause mucous production, which limits airflow. Allergic rhinitis affects the nose, pulmonary edema is the buildup of fluid in the lungs, and laryngitis is the inflammation of the larynx (vocal cords).

Snellen visual acuity screening should be performed

at a distance of 20 feet from the chart. Rationale Developed by Dutch eye doctor Hermann Snellen in the 1860s, there are many variations of the Snellen eye chart. In general, the charts show 11 rows of capital letters. They are designed for vision testing at a distance of 20 feet away from the chart. The Snellen test checks for distance visual acuity. The test is performed by having the patient read letters on a chart from a distance and determining the smallest row of letters the patient can read. Being seated or having the chart placed below eye level does not effect the outcome of this test. It would not be used at 14-16 inches from the patient.

A patient presents complaining of pain in the chest, left arm and shoulder. The results of the diagnostic tests indicate the coronary arteries are blocked with plaque. Which of the following conditions is this patient most likely to have?

atherosclerosis Rationale Atherosclerosis is the abnormal thickening of the walls of the arteries due to fatty deposits (atheromas) of cholesterol on the arterial inner walls. These thicken, forming plaques that narrow the vessel channel (lumen) and impede blood flow. Scarring and calcification make the walls less elastic, raising blood pressure. Peripheral vascular disease affects the blood vessels outside of the heart and especially those vessels supplying the extremities. Deep vein thrombosis is a condition marked by the formation of a thrombus within a deep vein (as of the leg or pelvis) that may be asymptomatic or be accompanied by symptoms (as swelling and pain) and that is potentially life threatening if dislodgment of the thrombus results in pulmonary embolism. Orthostatic hypotension, also called postural hypotension, is a form of low blood pressure that happens when you stand up from sitting or lying down.

An active preschool patient with thick nasal secretions is mouth-breathing due to congestion. The medical assistant should select which of the following methods for measuring body temperature?

axillary Rationale The axillary (under the arm pit) method of measuring body temperature is the least invasive, most reliable method for obtaining a temperature in this situation. A tight seal may be difficult to achieve around the mouth for this patient due to the congested mouth-breathing. Temporal and/or rectal methods are inadvisable for active children because it is difficult to get them to sit still long enough to obtain an accurate temperature.

Which of the following methods of measurement does the medical assistant select when obtaining a temperature on a crying, fearful, and anxious toddler?

axillary Rationale The medical assistant would use the axillary method when obtaining the temperature because it will provide the most accurate results in the least invasive manner. This method is the least likely option to further startle the child. A temporal reading would be nearly impossible to obtain since the patient is an active toddler who is already crying and fearful. An oral temperature would not be accurate since the toddler is crying (no seal would be created around the mouth and the temp would be inaccurate). A rectal temperature could provide an accurate reading, but would be more invasive and upsetting to an already distraught patient.

The branch of medicine which specializes in the study and treatment of patients with morbid obesity is

bariatrics. Rationale The prefix bar- means weight, and the -atri refers to the treatment of, therefore bariatrics is the study and treatment of morbid obesity. Gastroenterology is the study of the digestive system, endocrinology is the study of the endocrine system, and psychiatry study of mental disorders.

Which of the following valves separates the left atrium from the left ventricle?

bicuspid Rationale The bicuspic, also called mitral valve. It allows blood to flow one way only, from the left atrium to the left ventricle (prevents back flow or leakage). The pulmonary valve (sometimes referred to as the pulmonic valve) is the semilunar valve of the heart that lies between the right ventricle and the pulmonary artery and has three cusps. The tricuspid valve, or right atrioventricular valve, is on the right dorsal side of the mammalian heart, between the right atrium and the right ventricle. The aortic valve is the semilunar valve between the aorta and the left ventricle of the heart that prevents the blood from flowing back into the left ventricle.

Cystitis refers to inflammation of which of the following structures?

bladder Rationale The prefix cyst/o refers to the urinary bladder and the suffix -itis means inflammation, therefore cystitis is the inflammation of the bladder. Inflammation of the intestines is referred to as irritable bowel syndrome, the inflammation of the nerves is referred to as neuritis, and colitis is the inflammation of the colon.

Which of the following conditions is known as an early warning sign for skin cancer?

change in skin appearance and/or sore that does not heal Rationale Skin cancers begin as a precancerous lesion and develop into cancer with skin changes. An abnormal shape, irregular border, uneven color, change in size, or elevation on skin are common characteristics of cancerous lesions.

During a treadmill stress test which of the following vital signs should be monitored most frequently?

blood pressure Rationale During a treadmill stress test, blood pressure should be taken before, during, and after. Blood pressure should be taken every 3-5 minutes during the test and several post-exercise blood pressure tests must be performed once the test is complete. The respiration, pulse, and pulse oximetry are taken but not as frequently.

The suffix -emia means

blood. Rationale The suffix -emia refers to blood. Ex. anemia = lack of healthy red blood cells (hemoglobin).The suffix -itis means inflammation or infection, the suffix for excessive bleeding is -rrhage and the suffix for paralysis is -plegia.

Which of the following arteries is most commonly used for blood pressure measurement?

brachial Rationale Brachial artery - located in the antecubital region of the elbow. B/P is the measurement of force (pressure) of circulating blood against the walls of the artery. B/P is documented as Systolic (as the heart beats)/ Diastolic (as the heart relaxes between beats). EX. Average resting B/P for a young adult is at or below 120/80 mmHg (millimeters of mercury).

An abnormally slow respiratory rate is described as

bradypnea. Rationale The prefix brady means slow, therefore bradypnea is an abnormally slow respiratory rate. Dyspnea means difficulty breathing, orthopnea means difficulty breathing while lying down, and eupnea means normal breathing.

The central nervous system is made up of the

brain and spinal cord. Rationale The central nervous system (CNS) is made up of the brain and spinal cord. The heart is part of the cardiovascular system, the lungs are part of the respiratory system, and the kidneys and urinary bladder are with the urinary system. The pancreas and gallbladder are parts of the digestive system. Previous Question

Carcinoma refers to which of the following conditions?

cancer Rationale The term carcinoma means cancer, and does not refer to obesity, diabetes, or heart disease.

Which of the following structures is located at the distal end of the esophagus?

cardiac sphincter Rationale The lower esophageal sphincter is also referred to as the cardiac sphincter - it is a circular muscle that opens to allow food and fluids to enter the stomach and closes to prevent backward flow of gastric juices (reflux). The pyloric sphincter is located at the outlet of the stomach, allowing food to pass into the duodenum (small intestine). The common bile duct carries bile from the liver and gallbladder to the duodenum (small intestine). The ileocecal valve separates the small intestine from the large intestine in the lower GI system.

Which of the following arteries circulates blood flow to the brain?

carotid Rationale The carotid arteries are located on each side the lateral neck. They circulate blood flow to the brain, and are the most common site used to check for the presence of a pulse. The dorsalis pedis artery is in the foot, the popliteal artery is behind the knee, and the posterior tibialis is located by the ankle, and none of these arteries circulate blood flow to the brain

The ECG technician needs to obtain vital signs on an emaciated 86 pound geriatric female with an oxygen mask prior to obtaining her ECG. Which of the following equipment should the technician have on hand? (Select the two (2) correct answers.)

child sphygmomanometer, temporal thermometer Rationale Vitals are not always obtained before a standard ECG, but are warranted in this case. The child sphygmomanometer is appropriate to use since the smaller cuff would be used on a patient this small. A temporal thermometer would be selected over an oral thermometer in this instance since the patient is wearing an oxygen mask. The pulse oximeter will be utilized to obtain that vital information.

Which disorder involves the presence of stones in the gallbladder?

cholelithiasis Rationale Cholelithiasis is when there are small, hard deposits that form in the gallbladder due to a high concentration of bile (cholesterol). Cystitis is the inflammation of the bladder, cholecystitis is the inflammation of the gallbladder, and urolithiasis is when stones form anywhere in the urinary tract.

Which condition involves inflammation, scarring, and poor function of the liver?

cirrhosis Rationale Cirrhosis usually occurs in the final stages of chronic liver diseases and refers to the inflammation, scarring, and poor function of the liver. The suffix -itis is defined as inflammation of an organ therefore all responses describe inflammation of a part of the body. Colitis is defined as inflammation of the colon. Cholecystitis is inflammation of the gallbladder. Diverticulitis is inflammation of small pouches that line the intestine.

The lay term "collar bone" is

clavicle. Rationale The clavicle, lay term "collar bone", functions to support the shoulder, and helps to balance upper body weight. The scapula is either of a pair of large triangular bones lying one in each dorsal lateral part of the thorax, also known as the "shoulder blade". The sternum is a bone in the chest also called "breastbone", a long flat bony plate shaped like a capital T located anteriorly to the heart in the center of the thorax (chest). The adult spine is made up of approximately 24 bones (vertebrae) stacked on top of each other from the bottom of the skull to the pelvis.

Which of the following conditions is congenital?

cleft lip and palate Rationale A cleft lip and palate is an abnormal formation of an infant's upper lip and roof of mouth occurring during the 6-11th week of pregnancy and is congenital. Most hyperlipidemia is caused by lifestyle habits or treatable medical conditions. Many risk factors (genetic, environmental) can increase the chance of developing breast cancer, but it is not yet known exactly how some of these risk factors cause cells to become cancerous. The bacteria that cause tuberculosis are spread from one person to another through tiny droplets released into the air via coughs and sneezes.

The medical assistant is preparing the exam room for the next patient who will require urinary catheterization. The primary purpose for this procedure is to

collect a sterile urine specimen. Rationale A small, sterile tube is passed through the urethra to the bladder to obtain a specimen, free of contaminants, which will be collected in a sterile container. Urinary catheterization may also be ordered to test for urine residuals, or empty the bladder for a patient who is unable to void or incontinent.

The patient is not able to distinguish certain numbers on an Ishihara test. This indicates a problem with which of the following?

cones Rationale The Ishihara test is used to determine color blindness by means of a series of cards each having colored dots that form one pattern to the normal eye and a different pattern to the eye that is color-blind. In the anatomy of the Eye; the cones are responsible for color vision, the rods are responsible for the black and white and night vision. Both the rods and cones are part of the inner layer of the eye. The pupil is the opening in the center of the iris through which light enters the eye. The sclera (from the Greek skleros, meaning hard), also known as the white of the eye, is the opaque, fibrous, protective, outer layer of the eye containing collagen and elastic fiber.

The patient exhibits signs of anaphylaxis following an allergy injection in the medical office. The medical assistant should anticipate administration of

epinephrine. Rationale Anaphylaxis is a potentially fatal systemic (whole body) allergic reaction that can cause a multitude of symptoms, including shock, anxiety, sudden hypotension, narrowing airways, nausea, vomiting, itching and hives. Treatment for anaphylaxis involves an epinephrine injection, which can immediately reverse the life-threatening symptoms.

The medical assistant is caring for an older adult patient who is being directly admitted to the hospital for right lower lobe pneumonia. Which of the following findings are most likely for this patient? (Select the three (3) correct answers.)

cough, fatigue, dyspnea Rationale The patient would most likely present with a cough, fatigue, and dyspnea (trouble breathing). Hyperglycemia and/or a hive-like rash are not expected findings with pneumonia.

Referring to the Illustration, identify number 86 (shouldercap muscle)

deltoid Rationale Basic anatomy, rationale not applicable.

A medical assistant is providing information about the BRAT diet to a patient. The BRAT diet is indicated for what condition?

diarrhea Rationale BRAT=Bananas, Rice, Applesauce, and Toast. Recommended to act as a binder to ease digestion. Diet restriction is usually not necessary. Milk, carbonated drinks, caffeine, fruit juices containing sugar, and/or spicy foods should be avoided during the acute phase to prevent bloating, pain, or worsening diarrhea.

Which of the following represents the pressure of the heart as it relaxes between beats?

diastolic pressure Rationale Diastolic pressure (bottom number) - produced by the closure of the aortic and pulmonic valves (pressure when the heart is relaxed). Systolic pressure (top number) - produced by the force of blood in the arteries as the heart beats.

Which of the following symptoms is the patient diagnosed with benign prostatic hyperplasia most likely to report?

difficult-to-start urine stream Rationale Benign prostatic hyperplasia is when the prostate gland (located behind the urinary bladder), is enlarged and this could cause difficult urination due to the urethra being compressed, which is restricting normal urine flow. Nausea and vomiting, diffuse joint pain, and vomiting and diarrhea are not typically seen with benign prostatic hyperplasia.

Metatarsals are part of the

foot. Rationale Basic anatomy, rationale not applicable.

Which of the following is a common symptom of Alzheimer's dementia?

disorientation Rationale Alzheimer's dementia is a progressive deterioration of brain function caused by the destruction of cells (neurons) related to a build-up of amyloid plaques, which affect memory, language, thinking processes, and behavior. When caring for the patient, maintain a calm presence and include the patient in decision making when possible. Use a gentle, caring tone and simple terms in redirecting/orientating the patient to person, place, or time.

Which of the following symptoms reported by the patient should the medical assistant expect while performing a routine ear irrigation? (Select the three (3) correct answers.)

dizziness , itching, tingling sensation Rationale The patient should experience some itching in the ear, as well as a tingling sensation when the ear is being irrigated. Sometimes patients may feel dizzy during this procedure. Ear irrigation should not cause nausea or a headache. If these symptoms do appear, the physician should be notified immediately.

A patient calls the medical assistant complaining of painful intercourse. Which of the following is the correct medical term for this complaint?

dyspareunia Rationale Dyspareunia is the medical term for painful intercourse. Dysmenorrhea is a difficult or painful menstruation, and endometriosis is a condition within the uterine wall, metrorrhagia is a bleeding from the uterus, without having to do with menstruation.

The term meaning difficulty swallowing is

dysphagia. Rationale The term dysphagia means difficulty swallowing. Dystocia refers to a difficult labor and/or birth, dyspnea is difficulty breathing, and dystonia is when the muscles contract uncontrollably.

The medical assistant is taking a history on a patient with a urinary tract infection. Which of the following symptoms is the patient most likely to report?

dysuria Rationale A urinary tract infection is caused by bacteria entering the urethra and moving through the urinary tract. The most common symptoms include pain and burning during urination (dysuria), urgency, frequency, flank pain, foul-smelling, cloudy urine, and fever. Stress incontinence and enuresis are also both related to urination. Stress incontinence means urine leaks during exertion and enuresis is defined as a chronic problem in which a person cannot control their urination. Dysphasia is a condition in which the patient has difficulty speaking.

Which of the following conditions involves growth of a fertilized ovum outside the uterus?

ectopic pregnancy Rationale An ectopic pregnancy, also referred to as an Extrauterine pregnancy, occurs when a fertilized egg implants outside of the uterus. A pregnant woman who develops high blood pressure and protein in the urine is known to suffer from preeclampsia. Placenta Previa occurs when the placenta is located too close to the cervix, and pelvic inflammatory disease is the result of a woman's fallopian tubes, uterus, and ovaries being inflamed.

Which of the following conditions describes tissue swelling?

edema Rationale The term edema refers to the accumulation of fluid in the tissues, which causes swelling. Erythema is a term describing redness, pruritis is another word for itching, and epistaxis refers to a nosebleed.

Which of the following structures is composed of the visible, outermost layer of skin?

epidermis Rationale The skin is made up of three main layers: epidermis, dermis, and subcutaneous tissue. Four sub-layers within the epidermis (outside, top layer of skin): stratum corneum, stratum lucidum, stratum graunulosum, and stratum germinativum.

Which of the following drugs is expected to be stocked in medical office emergency code cart inventory?

epinephrine Rationale Most important drug kept in the crash cart - used in emergent conditions such as cardiac arrest (increases b/p and hr) or anaphylaxis (potent bronchodilator, reducing bronchospasm). Epinephrine works to increase cardiac output by increasing peripheral resistance through vasoconstriction.

Which of the following medical terms describes redness?

erythema Rationale The prefix erythro means red and the suffix -ema means abnormal condition, therefore the term erythema describes redness. Jaundice refers to a yellowish color, cyanosis refers to a bluish color, and pallor is a term that means pale.

Which of the following blood cells are formed in the bone marrow and transport oxygen?

erythrocytes Rationale Erythrocytes - a red blood cell that (in humans) is typically a biconcave disc without a nucleus containing the pigment hemoglobin, which imparts the red color to blood, and transport oxygen and carbon dioxide to and from the tissues. Thrombocytes - also known as platelets and are crucial to normal blood clotting. Leukocytes - or leucocytes, are the cells of the immune system that are involved in protecting the body against both infectious disease and foreign invaders. Monocytes - a type of white blood cells (leukocytes) which are the largest of all leukocytes and are part of the innate immune system

The medical assistant smells smoke at the work station. Upon investigation, he discovers a small fire in the trash receptacle by the desk. He should first

evacuate the immediate area. Rationale Using the RACE acronym -- RESCUE is the first thing the medical assistant should do, which includes evacuating the immediate area and making sure everyone is safely away from the fire. The next thing to do is to raise an ALARM by either pulling the fire alarm or calling 911 if the fire is not under control. Third, the fire needs to be CONTAINED; close all doors and windows that can be safely reached to make sure the fire does not spread. Lastly, EXTINGUISH the fire.

When observing a patient's range of motion of the right ankle, which term describes the ability to point their toes?

extension Rationale The range of motion (ROM) of a joint from full extension to full flexion (bending) measured in degrees like a circle. Abduction is the movement of an extremity on a transverse plane away from the axis or midline, where the axis lies on the frontal and sagittal planes, Adduction is the movement of a limb or other body part, usually on a transverse plane, toward the axis or midline—medial plane—of the body and Flexion is the act of bending a joint or limb in the body by the action of flexors.

Which of the following symptoms should the medical assistant expect to see in a patient with conjunctivitis?

eye redness Rationale Also known as "pink eye" - inflammation of the conjunctiva causing redness in the sclera, drainage (yellow crust), watery eyes, itching, burning. The conjunctiva is the thin clear tissue that lies over the white part of the eye and lines the inside of the eyelid. Pinkeye has a number of different causes, including: Viruses, Bacteria (such as gonorrhea or chlamydia), Irritants such as shampoos, dirt, smoke, and pool chlorine and Allergies, like dust, pollen, or a special type of allergy that affects some contact lens wearers. Pinkeye caused by some bacteria and viruses can spread easily from person to person, but is not a serious health risk if diagnosed promptly. Pinkeye in newborn babies, however, should be reported to a doctor immediately, as it could be a vision-threatening infection.

The patella is connected to the

femur. Rationale Basic anatomy, rationale not applicable.

Increasing which of the following nutrients in the diet is recommended to treat constipation?

fiber Rationale Fiber-rich foods keep bowel movements regular and decrease intestinal inflammation. The bulk and soft texture of fiber helps to prevent dry, hard stools. Fruits, vegetables, and whole grains are healthy sources of dietary fiber, and will help treat constipation. Protein, fat, and calcium do not have the ability to relieve constipation the way fiber does.

Which of the following bony structures is located in the lower extremity?

fibula Rationale The tibia and fibula are long bones in the lower leg that extend from knee to ankle. The sternum is a bone in the chest, the sacrum is a bone at the bottom of the spine, and metacarpals are bones in the hand.

The medical term meaning bending of a body part is

flexion. Rationale The word flexion means to bend. Tension means to keep something tight, pronation means to rotate the hand so the palm is facing downward, and the weight is placed on the inside of the foot, and supination is the opposite of pronation, so the palm would be facing up and the weight would be placed on the outside of the foot.

Which of the following indicators is considered a positive skin reaction to scratch allergy testing? A. excessive bleeding B. formation of a red wheal C. bruised appearance D. hemangioma formation

formation of a red wheal Rationale Hives, which are small, round raised areas on the skin (a.k.a. wheals; often red or white in color), are triggered by the body's inflammatory response (a.k.a. reaction). Itching may also be present. The purpose of allergy testing is to determine specific substances that cause an allergic reaction. Excessive bleeding and/or bruising (hemangioma formation) can indicate issues outside the immune response.

If the physician asks the medical assistant to place the patient in an anatomic position, the medical assistant should position the patient standing up straight and facing

forward, arms at the side with palms turned toward the front. Rationale The anatomical position is the erect position of the body with the face directed forward, the arms at the side, and the palms of the hands facing forward, used as a reference in describing the relation of body parts to one another.

The abbreviation meaning fracture is

fx. Rationale The abbreviation for fracture is fx. Tx is the abbreviation for treatment, sx is the abbreviation for surgery and can also be used as an abbreviation for symptoms. Rx is the abbreviation for prescription.

Cholecystitis is an inflammation of the

gallbladder. Rationale Choleycystitis is the inflammation of the gallbladder. The gallbladder aids in digestion of fats and concentrates the bile that is produced by the liver. Inflammatory bowel disease is the collective name for the inflammation of the intestines, splenomegaly is an inflamed or enlarged spleen, and pancreatitis is the inflammation of the pancreas.

The mouth, stomach, liver, and pancreas are parts of which of the following body system?

gastrointestinal Rationale The GI (gastrointestinal) system includes the upper and lower digestive systems, including the mouth, stomach, liver, and pancreas. The reproductive system includes the sex organs, the sensory system is included in the nervous system, and the hepatic portal system deals with the liver.

Which of the following sexually transmitted infections is caused by human papillomavirus (HPV), associated with cervical cancer?

genital warts Rationale The STI HPV is also known as genital warts. The immunization (Gardasil) is available to protect against the 4 most common types of HPV that may lead to cervical or other types of cancer (there are over 100 types of HPV). The AAP and CDC recommend this vaccine be given to girls at age 11-12, before sexual activity begins and prior to exposure of HPV. Symptomatic treatment is available, but no cure exists if HPV is contracted.

When obtaining a temperature reading using a tympanic thermometer, in which manner should the medical assistant move the pinna of a five-year-old patient?

gently pull up and back Rationale To straighten the ear canal shape to gain the best access to the tympanic membrane (ear drum) for an accurate reading would be achieved by gently pulling the pinna up and back. For smaller patients <3 years old, gently pull the ear lobe straight down and slightly back to gain the best position of the TM.

Which of the following conditions indicates infection and/or irritation of the gums?

gingivitis Rationale Inflammation of the gum tissue (gingivae) within the oral cavity is called gingivitis. Stomatitis and colitis refer to inflammation of the oral cavity and the colon, respectively. Thrush is a type of yeast infection that typically occurs in the mouth.

The prefix -adeno refers to which of the following body structures?

gland Rationale The prefix aden/o refers to the gland. Ex. Adenopathy refers to enlargement of a lymph node gland. The prefix for nerve is neuro, musculo is the prefix for muscle, and the prefix for joint is arthro.

Which term describes the presence of sugar in the urine?

glucosuria Rationale The prefix gluc/o means sugar, and the suffix -uria refers to urine, therefore the term glucosuria refers to the presence of sugar in the urine. Ketonuria is the presence of ketones in the urine, which are substances that are formed when fat is broken down in the body to make energy. Glycogenolysis is when glycogen is broken down to make glucose, and polycystic is a term referring to multiple cysts.

Referring to the Illustration, identify number 90. (main butt muscle)

gluteus maximus Rationale Basic anatomy, rationale not applicable

Referring to the Illustration, identify number 89. (Hip muscle)

gluteus medius Rationale Basic anatomy, rationale not applicable.

Metacarpals are part of the

hand Rationale Basic anatomy, rationale not applicable.

During an office visit a patient asks the medical assistant what his BMI is. Which of the following information should the medical assistant measure to calculate BMI?

height and weight Rationale Height and Weight are used for the formula to get BMI, which tells if the PT is obese. BMI = WT /HT

Which of the following conditions is exhibited by paralysis on one side of the body?

hemiplegia Rationale The prefix hemi means half and the suffix -plegia refers to paralysis of something, therefore hemiplegia is the paralysis of one side of the body. Paraplegia is the paralysis of the lower extremities, hemorrhagia is a term referring to a large amount of bleeding, and quadriplegia is the loss of function of all of the extremities.

Painful, dilated veins occurring in the lower portion of the rectum or anus are characteristic of which of the following conditions?

hemorrhoids Rationale Hemorrhoids are painful, dilated veins that appear in the rectum or anus, and are caused by venous congestion or increased pressure in or around the anal canal. They may occur with straining to move a stool, pregnancy, aging, or chronic constipation. Polyps are growths that occur in the body that can become cancerous. Ulcers are sores that can be found in the lining of the intestine or stomach, and a fissure is a tear or separation in the skin that causes bleeding.

Inflammation of the liver is referred to as which of the following conditions?

hepatitis Rationale The prefix hepat/o means liver, and the suffix -itis means inflammation, therefore hepatitis is the inflammation of the liver. Encephalitis is the inflammation of the brain, pyelonephritis is the inflammation of the kidneys, and cholecystitis is the inflammation of the gallbladder.

What is a major factor that contributes to an increased percentage of greenstick fractures in children?

high cartilage content in bones Rationale Children have a high cartilage content in their bones, which contributes to an increased percentage of greenstick fractures. A greenstick fracture is an incomplete break, so the bone bends but doesn't completely break. The lack of adipose tissue, increased metabolism, and wasting of muscle due to inactivity aren't factors that lead to more greenstick fractures in children.

Which of the following is a long bone?

humerus. Rationale Basic anatomy, rationale not applicable.

What is the prefix meaning abnormally increased or excessive?

hyper- Rationale The prefix hyper- means high, or above normal. Ex. Hypercalcemia is high calcium in the blood. The prefix post- means after, hypo- means low, and pre- means before.

An adult patient with vital signs of T 37.0 C, P 72, R 18, B/P 192/102 mm Hg indicates which of the following conditions?

hypertension Rationale These vital signs align with a patient experiencing hypertension (a.k.a. high blood pressure). Both the systolic and diastolic readings are high. Postural hypotension (orthostatic hypotension) is low blood pressure resulting when a patient stands after sitting or lying down. Tachycardia refers to a high heart rate. A normal adult resting pulse rate is 60-100 beats per minute, so this patient is within normal limits at 72. Hypoxia translates to low oxygen levels and that was not indicated in these vitals.

The medical assistant measures the patient's blood pressure in the office. The reading is 220/88 mmHg. This indicates

hypertension. Rationale Normal Blood Pressure ranges for an adult: 100/60 to 119/79. Any readings above these would indicate hypertension (high blood pressure). Hypotension indicates low blood pressure. Bradycardia is a slow heart rate, whereas tachycardia is a fast heart rate.

The medical assistant measures a patient's b/p in the left upper arm at 188/110 mm Hg. This reading is consistent with

hypertension. Rationale This patient's blood pressure is above the normal reading, and this is referred to as hypertension. Hyperplasia is the increased cell production, tachycardia is increased heart rate, and hyperthermia is increased body temperature.

A patient with a blood pressure of 158/96 mm Hg is considered

hypertensive Rationale Normal adult blood pressure readings range from 100/60-119/79 mm Hg. Readings above this range would indicate hypertension (a.k.a. high blood pressure). Hypotension is commonly known as low blood pressure. Bradycardia indicates a slow heart rate, whereas tachycardia represents a fast heart rate.

Bowel sounds with reduced volume, tone, or regularity upon auscultation are described as

hypoactive. Rationale The prefix hypo means slow, therefore hypoactive refers to a reduced volume, tone, or regularity of bowel sounds upon auscultation. When bowel sounds are absent, it means there are none heard, when they're hyperactive it means the sounds are increased. Normoactive bowel sounds means the sounds are regular and normal.

A patient presents with frostbite of the phalanges. This indicates which of the following?

hypothermia Rationale Frostbite occurs when a person is exposed to extremely cold conditions and experiences hypothermia. Hypothermia involves the body temperature dropping very low. When the body is exposed to these cold conditions, skin and tissue damage is possible. This is known as frostbite. Hyperthermia refers to the body reaching very high temperatures and anaphylaxis is a serious allergic reaction. Vasovagal syncope refers to fainting when the body reacts excessively to something.

A patient has a pulse oximetry reading of 88% on room air. Which of the following conditions does this reading indicate?

hypoxia Rationale The hemoglobin in red blood cells is responsible for carrying oxygen in the body. This carrying capacity is measured by a pulse oximeter. In most circumstances, a normal oxygen saturation reading would fall between 95%-100%. Generally, a pulse oximetry reading under 90% would indicate low blood oxygen levels (a.k.a. hypoxia). Eupnea indicates a normal breathing, apnea is without breath, and arrhythmia describes an irregular heartbeat.

Which of the following conditions describes poor oxygenation?

hypoxia Rationale The prefix hypo means low or below the normal level, and the suffix oxia refers to oxygen, therefore the condition hypoxia describes poor oxygenation. Jaundice refers to a yellow coloring of the skin or eyes, hypothermia is a term used to describe an extremely low body temperature, and erythema is when the skin or mucous membranes have a reddish color.

Which of the following terms means surgical removal of the uterus?

hysterectomy Rationale The prefix hyster/o means uterus, and the suffix -ectomy means the surgical removal of, therefore a hysterectomy is the surgical removal of the uterus. An ileostomy is the surgical removal of the ileum, which is part of the small intestine. An oophorectomy is the surgical removal of the ovaries, and a uteroscopy is a device used to view the inside of the uterus.

The suffix -blast refers to

immature cells. Rationale The suffix -blast means immature. For example, osteoblasts are immature cells that work to produce bone formation. The suffix for fluid accumulation is -ous, -lysis is the suffix for cell destruction, and the suffix for cell rupture is -rrhexis.

Which of the following conditions involves the inability to control the flow of urine in the bladder?

incontinence Rationale Incontinence is the inability to control the flow of urine in the bladder. Dysuria is painful urination, polyuria is a large amount of urine, and oliguria is a small amount of urine.

The patient's temperature is 40C (104F). Which of the following signs and symptoms does the medical assistant anticipate, indicating the body's physiologic response to the fever?

increased heart rate, increased respiratory rate, chills Rationale The medical assistant would expect to see an increased heart rate, increased respiratory rate, and chills. These are all common body responses to a fever. Blood pressure changes are not necessarily directly associated with a fever, but could be associated with other issues a patient may be experiencing in addition to a fever.

A patient teaching plan to prevent osteoporosis includes

increasing dietary intake of calcium and vitamin D. Rationale Adequate calcium, vitamin D, and regular exercise are important for protecting bone health and strength. Too much caffeine and/or phosphorous intake (contained in carbonated beverages), can cause a decreased absorption of calcium in the body. Short chain fatty acids (a type of saturated fat) have shown to have anti-inflammatory properties and can inhibit the growth of colon cancer cells. Proper exercise including some weight bearing exercise can strengthen the bones, improve balance, and reduce risk of fracture. Not all fiber is good for vitamin D absorption. Insoluble fiber: the type of fiber, such as the kind in wheat bran, reduces calcium absorption. Previous Question

Applying a warm, moist pack to a soft tissue injury promotes healing by

increasing vascular circulation. Rationale Warm packs (heat sources) can help increase vascular circulation and are usually applied for short intervals. The warmth from a warm, moist pack improves mobility and relaxes muscles. The use of ice would constrict the blood vessels and is used on an injury with the potential to swell. Increasing skin elasticity or decreasing the flow of lymphatic fluid (associated with Edema) is not needed with this type of injury.

The presence of which of the following conditions is most likely to affect a patient's temperature when measuring vital signs?

infection Rationale Fever is an indicator of the body's immune response to infection and/or inflammation. Vital signs provide an objective measurement of a patient's overall health status. Accuracy of vital signs is important because the information can significantly affect patient care and treatment. Vital signs can help identify instances and trends that impact clinical (and pharmaceutical) treatment decisions.

The patient diagnosed with bursitis has which of the following conditions?

inflammation around muscles, joints, and tendons Rationale Bursitis is inflammation around the muscles, joints, and tendons. The suffix -itis refers to inflammation, and the suffix for pain is -algia, bleeding is -rrhagia, and destruction is -lysis.

Stomatitis refers to which of the following conditions?

inflammation of the mouth Rationale The prefix stom(a)- refers to the mouth, and -itis means inflammation, therefore stomatitis refers to the inflammation of the mouth. The narrowing of the esophagus is referred to as a stricture, hepatomegaly is the enlargement of the liver, and esophagitis is the inflammation of the esophagus.

A patient's chief complaint of difficulty falling asleep is described as which of the following conditions?

insomnia Rationale The suffix -somnia means sleep, and the term insomnia refers to difficulty falling asleep. Nocturia is the need to urinate at night, narcolepsy is excessive drowsiness, and sleep apnea is when a person stops breathing for periods of time while they're sleeping.

Which of the following body systems includes the skin, hair, and nails?

integumentary Rationale The integumentary system includes the skin, hair, and nails, and is the largest organ of the body. The word integument means "to cover and protect", and important functions of the skin include: protection, regulation, sensation, absorption, secretion. The lymphatic system is composed of lymph nodes, tonsils, spleen, adenoids, and thymus, and the main job of this system is to get rid of toxins in the body. The immune system's job is to protect the body against foreign objects and disease. The endocrine system is a group of glands that secrete hormones to help regulate the body.

The adult patient's respiratory rate is 18 breaths per minute, with no apparent distress. This finding

is within normal limits. Rationale The normal range for an adult is 12-20 breaths per minute, so this finding would be within normal limits and would not need to be reported to the physician. Tachypnea would be a respiratory rate above 20 breaths per minute, and apnea is when breathing is stopped.

A prolonged lack of oxygen to the heart muscle leads to

ischemia. Rationale When the heart muscle is deprived of oxygen for an extended period of time, ischemia occurs. Anemia is associated with a lack of red blood cells, resulting in low hemoglobin levels. A thrombus is a blood clot, and phlebitis indicates inflammation of the veins.

Which of the following terms describes a yellowish-appearance to the skin?

jaundice Rationale The term jaundice describes a yellowish-appearance of the skin, and is caused by high bilirubin levels in the blood. Cyanosis is a bluish color, pallor is another word for pale, and erythema is a reddish color.

On the image locate the maxilla.

jaw Rationale Basic anatomy, rationale not applicable.

Which of the following structures contain the vocal cords?

larynx Rationale Basic anatomy, rationale not applicable.

A pediatric patient presents to the clinic with a barking cough and high-pitched breathing. These signs indicate to the medical assistant that which of the following anatomical structures is obstructed? A. larynx B. bronchial tree C. soft palate D. trachea

larynx Rationale Stridor (barky, seal-like cough) is heard on inspiration and is due to the blockage of the larynx. These abnormal breath sounds are often associated with Croup, which primarily affects the pediatric population. The obstruction of the bronchioles occurs in asthma, and sleep apnea occurs when the soft palate is collapsed. An obstruction of the trachea would not cause the barking cough or the high-pitched breathing, because the vocal cords that are causing the abnormal sounds are in the larynx and not the trachea.

Referring to the Illustration, identify number 88. (back of ribcage muscles)

latissimus dorsi Rationale Basic anatomy, rationale not applicable.

Which of the following procedures involves the process of cleaning out the contents of the stomach through a nasogastric tube?

lavage Rationale Lavage is the irrigation of the stomach through a nasogastric tube. Gavage is the input of food into the body through a tube, a cleansing enema is used to clean out the rectum and relieve constipation, and a lumbar puncture is also known as a spinal tap and is when a needle is inserted into the back to remove fluid.

The medical assistant is obtaining a blood pressure measurement on a patient with a right-sided mastectomy. At which of the following sites should the medical assistant take the blood pressure?

left upper arm Rationale If possible, the blood pressure should be measured as close to the heart as possible, while also avoiding the right arm in this patient due to their mastectomy. Obtaining the blood pressure on the right arm would increase the risk of post-operative swelling and pain of the arm and/or incision. The left arm should be used since it is closer to the heart than either of the legs.

Which of the following chambers of the heart is responsible for pumping blood through the aorta to the body?

left ventricle Rationale The left ventricle is the main pumping chamber of the heart, and is in charge of pumping blood through the aorta to the body. Blood is pumped into the right atrium, then to the right ventricle, then to the lungs, and then is oxygenated and pumped into the left atrium, and finally to the left ventricle before going to the body.

When setting up the room for a gynecological exam, the medical assistant will position the stirrups in which of the following ways?

level with the exam table and pulled out approximately 1 foot from the edge of the table Rationale To properly assist the patient into the lithotomy position, which is the position of choice for this exam, the stirrups need to be level with the exam table and pulled out approximately 1 foot from the edge of the table. This allows the patient to be as close as possible to the gynecologist.

The physician orders the administration of oxygen to a patient. The medical assistant should document the oxygen flow rate as

liters per minute. Rationale Oxygen flow rate is measured in liters per minute.

The patient experiencing dyspnea is placed on oxygen via nasal cannula. The medical assistant should document the flow rate as

liters per minute. Rationale The usual oxygen flow meter is between 0-15 liters per minute. The flow should be adjusted per physician's order, typically initiated for an adult between 2-6 LPM via nasal cannula.

The physician requests for the medical assistant to assist the patient into the correct position for a pap smear. The medical assistant should assist the patient into which of the following positions?

lithotomy Rationale Lithotomy is a position in which the patient's knees are bent with feet flat on a table or in stirrups. This position is used to maximize visibility for vaginal, anal, rectal, or perineal area exams. Sims positioning has the patient on a side with the top knee bent. Knee-chest has a patient pull knees to chest with the bottom in the air. Fowler's position is a commonly recognized position in which a patient lies in a supine position with the head elevated 20 to 30 inches.

A patient presents to the office for removal of an IUD. Which of the following exam positions should the medical assistant help prepare the patient to assume on the exam table?

lithotomy Rationale When a patient is in the Sims position, he or she lies on one side with one knee bent. The Trendelenberg positions the patient head down while elevating the feet (this position is widely used to manage hypotensive patients, but the evidence to support its effectiveness is suspect). A knee-chest position has a patient prone on the knees with chest on the exam table. Lithotomy is a positions a patient on the back with knees bent and feet up in stirrups.

The physician instructs the medical assistant to prepare for a simple laceration repair of a patient's superficial leg injury. Which of the following equipment does the medical assistant obtain to assist the physician with this procedure? (Select the four (4) correct answers.)

local anesthetic supplies, sterile gloves , suture tray, Mayo stand Rationale A typical laceration repair setup includes: Mayo stand, sutures of specified type, brand, and size, waste container/plastic bag, adjustable stool, overhead light turned on, local anesthetic supplies of specified type, towel pack, 4x4 gauze sponge pack, forceps, two pairs of sterile gloves for physician in correct size, patient drape, needle pack, surgical/ suture tray including suture materials, sterile basin and normal saline/ irrigation supplies. Obtaining a set of blood cultures is not indicated.

The radius, ulna, femur, and tibia are referred to as

long bones. Rationale The radius, ulna, femur, and tibia are located in the upper and lower extremities and are all referred to as long bones. The spinous process is in the back, cartilage is a tissue found all over in the body, and the vertebrae are in the spine.

The term oliguria is used to describe which of the following conditions?

low urine output Rationale The prefix oligo- means few or scant, therefore olgiuria means low urine output. Cystitis refers to the inflammation of the bladder, frequent urination is called polyuria, and renal calculi are also referred to as kidney stones.

The femur is located in the

lower extremity. Rationale Basic anatomy, rationale not applicable.

Which of the following diseases is caused by exposure to carcinogens?

lung cancer Rationale Carcinogens are agents that may cause all types of cancer by working to change the activity of a cell and damage the DNA (causing abnormalities and unhindered cell division). Lung cancer can be caused by exposure to carcinogens that are in tobacco, radon, and asbestos. Rheumatoid arthritis (RA) and systemic lupus erythematosus (SLE) are autoimmune disorders in which the body mounts an immune response to itself, effectively attacking it's own tissues. The exact cause of autoimmune disorders is not known. Infectious mononucleosis is a viral infection caused by the Epstein-Barr virus (EBV).

The medical term meaning a tumor located in the lymphatic system is

lymphoma. Rationale The suffix -oma means a tumor or growth, therefore a lymphoma would be a tumor in the lymphatic system. Lymphadenitis is an inflammation of a lymph node, hemangioma is the accumulation of blood vessels somewhere in the body, such as the skin or organs, and an osteosarcoma is a tumor that occurs in the bone.

The physician is referring a patient to a dermatologist for a mole that has suddenly changed in appearance. A positive pathology report would most likely indicate

melanoma. Rationale Melanomas often resemble moles; some develop from moles. Lymphoma is a form of cancer that affects the immune system, Adenoma is a type of non-cancerous tumor or benign that may affect various organs, and Osteosarcoma is a cancerous tumor in a bone.

The time period when menstruation begins describes which of the following conditions?

menarche Rationale Menarche is when the menstrual period begins. Menopause is when menstruation stops, amenorrhea is the absence of menstruation, or when a woman misses more than one menstrual period, and gestation is a term referring to a baby being carried in the womb.

The time period when menstruation stops is described as which of the following conditions?

menopause Rationale Menopause is the transitional time period of a woman's life when ovarian function ceases and menstruation stops. Dysmenorrhea refers to the pain that women feel during their menstrual periods, menorrhagia means abnormally heavy bleeding during the menstrual period, and menarche refers to the onset of the menstrual period.

Ligaments, tendons, cartilage, joints, and bones are part of which of the following body systems?

musculoskeletal Rationale The human musculoskeletal system is an organ system that gives humans the ability to move using their muscular and skeletal systems. The musculoskeletal system provides form, support, stability, and movement to the body. It is made up of the bones of the skeleton, muscles, cartilage, tendons, ligaments, joints, and other connective tissue that supports and binds tissues and organs together. The musculoskeletal system's primary functions include supporting the body, allowing motion, and protecting vital organs. The integumentary system comprises the skin and its appendages. The nervous system is composed of the brain, spinal cord and nerves. The endocrine system include the pineal gland, pituitary gland, pancreas, ovaries, testes, thyroid gland, parathyroid gland, hypothalamus, gastrointestinal tract and adrenal glands.

Which of the following is likely to produce an inaccurate pulse oximetry reading?

nail polish Rationale Pulse oximetry is a method of evaluating oxygen saturation of hemoglobin and pulse rate. The pulse oximeter utilizes a small probe that is put on the fingertip or earlobe. A light signal passes through the tissue to a photo detector. Nail polish will interfere with the light that passes through the tissue, and will lead to a false low measure of the oxygenated hemoglobin (if the polish is dark, it could fully block the signal). This is a noninvasive method of measuring the saturated hemoglobin, a useful screening tool for determining basic respiratory function. The presence of nail polish may affect the light sensor reading within the probe. To correct for this issue, place the pulse oximeter on a peripheral finger that is free of nail polish. Tachypnea (rapid breathing) and bradycardia (slow heart rate) would not hinder a pulse oximeter reading. Since the device is placed on the fingertip or ear lobe, a wrist tattoo would not cause an issue. However, a highly pigmented tattoo on a finger could impede the signal (so that finger would not be a good candidate for oximeter placement).

Which of the following structures is the site of initial entry of air into the respiratory tract?

nasal cavity Rationale The upper airways or upper respiratory tract begins with the nose and nasal passages, then the paranasal sinuses, the pharynx, and the portion of the larynx above the vocal cords. The lower airways or lower respiratory tract includes the portion of the larynx below the vocal cords, trachea, bronchi and bronchioles. The lungs are included in the lower respiratory tract and include the respiratory bronchioles, alveolar ducts, alveolar sacs, and alveoli.

The medical term describing tissue death is

necrosis. Rationale The term necrosis means premature death of living cells or tissue. Cyanosis is a bluish color that occurs when there is low oxygenation to the tissues, sclerosis is the stiffening of a body part, and cirrhosis is the scarring of the liver, which results in decreased liver function.

The medical assistant measures the patient's blood pressure at 178/108 (mmHg) during the check-in process. Which of the following actions should the medical assistant take?

notify the health care provider Rationale The normal adult blood pressure range is 100/60-119/79 mm Hg. A BP reading of 178/108 would be considered hypertension (high blood pressure). The health care provider would need to be notified because a BP that high could be harmful to the patient if left untreated. Nitroglycerin is administered when a client is experiencing chest pain, which the patient in this scenario is not. Assisting the patient into Trendelenburg position will not fix the blood pressure problem. Though it might be a contributing factor to overall patient health, calculating the patient's BMI isn't the best approach in this situation because knowledge of the BMI would not have an immediate affect on the hypertension issue.

The patient reports small amounts of urine excretion. Which of the following medical terms describes this condition?

oliguria Rationale A small amount of urine excretion is called oliguria. Hematuria is when there is blood in the urine, dysuria is difficulty urinating or painful urination, and anuria is when there is no urine being produced.

A patient with a tracheostomy has a surgical

opening of the throat. Rationale A tracheostomy is the formation of a surgically created opening in the windpipe (throat). An opening of the chest would be a thoracotomy or other procedure in which there is an opening in the chest wall. A repair to the throat or the chest would include the suffix -plasty.

A patient enters the ER with complaints of chest pain. After the patient is attached to the ECG machine, the patient will be monitored for which of the following while the ECG runs continuously?

oxygen levels Rationale A patient complaining of chest pain needs to be monitored very closely. An ECG machine is often attached in the ER and continuous oxygen levels are monitored to keep tabs on the oxygen saturation levels. The other vital signs will be taken at whatever intervals the staff deem appropriate for the care of this patient.

A pulse oximetry monitor measures which of the following?

oxygen saturation Rationale A pulse oximetry monitor is a noninvasive measurement of the oxygen saturation of a patient's arterial blood. Pulse pressure is the difference between the systolic and diastolic blood pressure, respiratory rate is measured by counting the patient's breaths, and oxygen flow rate is the set amount of oxygen that a patient is receiving while wearing a nasal cannula or oxygen mask.

The suffix -algia refers to which of the following clinical symptoms?

pain Rationale The suffix -algia refers to pain. Example. Fibromyalgia is a chronic condition of widespread pain in the muscles and connective tissues. The suffix for bleeding is -rrhage, -narc is the suffix for numbness, and -rrhea is the suffix for abnormal flow.

Which of the following conditions describes dysuria?

painful urination Rationale The prefix "dys" means difficult or painful. The suffix "uria" refers to urine. Therefore, the term "dysuria" means painful urination. The term for blood in the urine is hematuria. An individual experiencing frequent urination would urinate more often than an average individual. Mucous in the urine is not necessarily an abnormal indication.

The provider in the photo is using which of the following methods to examine this patient? (rubbing with two stacked hands)

palpation Rationale The process of feeling with the fingers or hands for diagnostic purposes during a physical exam is called palpation. Auscultation is the process of listening to a certain area of the body with a stethoscope. When a provider does an inspection of a patient, he/she will look at the specific body part(s) to check for abnormalities before they move on to percussion or palpation. Percussion has medical providers tapping on a part of the body using a percussion hammer or their fingers.

Which of the following accessory organs produces digestive enzymes and secretes the hormone insulin?

pancreas Rationale The pancreas is a glandular organ in the digestive system and endocrine system that aids in digestion of food and blood glucose control. The liver has multiple functions, but its main function within the digestive system is to process the nutrients absorbed from the small intestine. The gallbladder stores and concentrates bile, and then releases it into the duodenum to help absorb and digest fats. Saliva from the salivary glands mixes with food to begin the process of breaking it down into a form the body can absorb and use.

A patient presents to the urgent care office reporting a burn that occurred a few minutes ago. The left wrist is painful, red, swollen and has blisters. How should the medical assistant record the severity of the burn on the patient's chart?

partial-thickness burn (2nd degree) to the left wrist Rationale All burns are evaluated on a depth and thickness scale. A burn is a type of injury to flesh or skin caused by heat, electricity, chemicals, friction, or radiation. Burns that affect only the superficial skin are known as superficial or first-degree burns. When damage penetrates into some of the underlying layers, it is a partial-thickness or second-degree burn. In a full-thickness or third-degree burn, the injury extends to all layers of the skin. A fourth-degree burn additionally involves injury to deeper tissues, such as muscle or bone.

The ischium is part of which of the following bony structures?

pelvis Rationale The ischium is part of the pelvis and helps form the posterior and inferior portion of the hip. The clavicle and scapula are both bones in the chest, and the phalanges are bones in the fingers and toes.

Which of the following bones supports and protects the lower abdominal organs?

pelvis Rationale The pelvis is a circular bone structure made up of the sacrum, coccyx, ilium, ischium, and pubis. When one or more of these bones are broken it is referred to as a pelvic fracture. The patella is also known as the "knee cap". The scapula is either of a pair of large triangular bones lying one in each dorsal lateral part of the thorax, also known as the "shoulder blade". The septum is a dividing wall or membrane especially between bodily spaces or masses of soft tissue, i.e. nasal septum or crural septum.

The older adult patient who is observed talking in the clinic waiting area begins to choke on a piece of hard candy and cannot speak. Which of the following actions should the medical assistant take immediately?

perform abdominal thrusts Rationale Since the patient is not experiencing heart issues, utilizing an Automated External Defibrillator and administering chest compressions are not warranted. Baseline vitals are not the primary concern, as the immediate need is clearance of the airway. A choking person might need intervention to assist in dislodging the blockage. Abdominal thrusts (a.k.a. the Heimlich Maneuver) are an effective way to assist in this situation. To administer abdominal thrusts, stand behind the patient while wrapping arms around the torso directly above the navel. Make a fist (thumb in) with one hand, cover it with the other, and thrust upward and inward with force sufficient enough to briefly lift the patient off his or her feet.

Which of the following measurements indicate the difference between the systolic and diastolic blood pressure?

pulse pressure Rationale Systolic pressure minus diastolic pressure = pulse pressure. This is the amount of pressure needed in the circulatory system to create the feeling of a pulse on palpation. Heart rate is the rate at which the heart is pumping out blood, body mass index is the measurement of the amount of fat a person has in their body, and the point of maximum impulse is where the heart rate is best heard.

The presence of which of the following is most likely to cause a false low reading on an oxygen saturation monitor?

pink nail polish Rationale An oxygen saturation monitor is a device that measures the amount of saturated hemoglobin in the tissue capillaries by transmitting a beam of light through the tissue to a receiver. It may be used on either the earlobe or the fingertip. This is a noninvasive method of measuring the saturated hemoglobin, a useful screening tool for determining basic respiratory function. The presence of nail polish may affect the light sensor reading within the probe. To correct for this issue, place the pulse oximeter on a peripheral finger that is free of nail polish. Jaundice is a medical condition with yellowing of the skin or whites of the eyes resulting from an excess of bilirubin (typically caused by obstruction of the bile duct). Anaphylaxis is an acute allergic reaction to an antigen to which the body has become hypersensitive (this would not influence this oxygen saturation monitor). An artifact would likely cause a reading error (or show no number instead of a low reading as is listed in this scenario). Black or dark nail polish could also block the signal.

The patient with a penetrating wound to the chest cavity is acutely at risk for which of the following complications?

pneumothorax Rationale A pneumothorax, also referred to as a "collapsed lung" is a build-up of air or gas that separates the lung from the chest wall and decreases lung expansion. When a wound penetrates the chest cavity, air can enter the area between the lung and the chest wall. Epistaxis is another word for a nose bleed, pneumonia is an infection of the lungs, and emphysema occurs when the little sacs in the lungs, known as alveoli, are damaged, which makes it difficult to breathe.

Which of the following conditions is most likely to cause varicose veins?

pregnancy Rationale Varicose veins are enlarged veins filled with an abnormal collection of blood, and can be caused by pregnancy, prolonged standing, prolonged bed rest, obesity, and aging. Menstruation, osteomyelitis, and water-loss stools would not cause varicose veins, because they would not cause the blood to pool in the veins.

Which of the following hormones is produced by the ovaries?

progesterone Rationale Progesterone is a hormone that regulates ovulation and menstruation, and is produced by the ovaries. Testosterone is produced by the gonads, oxytocin is produced by the hypothalamus, and prolactin is produced by the pituitary gland.

Which of the following clinical findings is common for an infant with pyloric stenosis?

projectile vomiting Rationale A gastric outlet obstruction caused by a progressive enlargement of the pyloric muscle affecting infants between birth and 5 months. Symptoms include forceful vomiting of most of feedings, weight loss, irritability, and dehydration - surgical correction is necessary (pyloromyotomy).

The medical assistant is caring for the patient in the clinic diagnosed with a right ankle sprain that occurred during a gymnastics tournament approximately three hours ago. Which of the following treatments are appropriate? (Select the four (4) correct answers.)

protect and rest the affected ankle, apply an ice pack to the ankle, wrap the affected ankle with a compression bandage, elevate the ankle at or above heart level Rationale The PRICE method is commonly used in instances such as this. PRICE: Protect, Rest, Ice, Compression, and Elevation. The patient should protect (immobilize and refrain from putting pressure on) the ankle from further injury and rest it. Ice the affected area, taking care to protect the skin from direct ice exposure. Compress the injury with an ace bandage wrap or something similar (avoid wrapping too tightly, which may obstruct blood flow). Elevate the injury above the level of the heart. These steps help to decrease/relieve pain, limit swelling and promote healing.

Which of the following medical terms describes a noncontagious inflammatory skin disease characterized by recurring reddish patches covered with silvery scales?

psoriasis Rationale Normal skin cells mature and die every 29-30 days. Psoriasis causes cells to mature and die every 3-6 days, resulting in skin sloughing off and a chronic buildup of silvery scaled skin. Alopecia is associated with balding. Keratosis is a buildup of skin cells whereas eczema is an idiopathic inflammatory skin disorder generally marked by itching and redness.

The adult patient presents to the clinic for a follow-up visit one week after a ruptured appendectomy. The medical assistant obtains the following measurements: T 36.9C (98.5F), P 110, R 28, B/P 88/56, O2 Sat 96%. Which of the following findings should be reported to the provider? (Select the three (3) correct answers.)

pulse, respirations, blood pressure Rationale The provider should be notified about the pulse, respirations, and blood pressure. A normal pulse range is 60-100, a normal respiratory rate is 12-20, and a normal blood pressure is 120/80. Therefore, all of these demonstrate abnormal, reportable findings. The temperature is normal, and the oxygen saturation is also normal

Providing high-quality chest compressions while performing CPR

pumps blood from the heart. Rationale Chest compressions are essential while administering CPR, doing the work of the heart muscle to generate blood flow to vital organs during cardiopulmonary arrest. When available, an automated external defibrillator (AED) can help restore normal heart rhythms in a patient experiencing cardiac arrest. To maintain open airway alignment, keep the patient in the supine position on a hard flat surface with the head tilted back and mouth open, head and neck aligned. Patients can aspirate gastric contents if the larynx is not functioning in an unconscious state.

The patient presents to the clinic with right great toe pain and soreness due to Gout. Which of the following types of dietary nutrients is most likely to worsen these symptoms?

purine-rich foods Rationale Purine-rich foods (dairy and protein) and beverages containing caffeinate or alcohol worsen symptoms and should be avoided. Fruits and vegetables (cherries, blueberries, and cabbage) contain substances that decrease uric acid levels causing inflammation. Foods rich in fiber also protect against gout symptoms.

While changing a patient's dressing, the medical assistant notices thick, yellow drainage with a foul odor coming from the wound. The wound exudate noted should be documented as

purulent. Rationale Purulent describes a thick drainage containing pus with a yellowish color to it, and is not a normal finding in a wound. Serous drainage is thin and clear in appearance, sanguineous drainage is fresh blood, and serosanguinous drainage is a pinkish color.

Which of the following terms describes a wound that is pus-producing?

pyogenic Rationale The prefix py/o referes to pus, and the suffix -genic refers to the formation of, therefore pyogenic means pus-producing. The term parietal refers to something that is attached to or is touching a wall or cavity, and percutaneous means the skin was punctured. When something is delivered through the parenteral route, it means it is not given through the intestinal tract, and is delivered somewhere else in the body, like through a feeding tube.

Which of the following is the pulse site most commonly used when taking a pulse rate for routine vital signs on the adult patient?

radial Rationale The most common site to take a pulse when gathering routine vital signs is the radial site, which is located on the wrist. The carotid site is typically used when determining whether a patient needs CPR or not. The brachial site is used when assessing blood pressure. The apical site is typically used as a more accurate pulse reading if the radial pulse is abnormal.

Referring to the image, the medical assistant will select which of the following sites to measure the heart rate when obtaining vital signs?

radial artery Rationale The radial artery is most easily accessed and palpated (inside of the wrist just below the thumb) for measurement. The cephalic vein runs from the hand to the shoulder and is typically used for blood samples. The brachial artery is used when measuring blood pressure. The basilic vein runs along the forearm and is not used to measure heart rate. Previous Question Next Question

The patient is scheduled for an abdominal CT scan to evaluate for appendicitis. The medical assistant instructs the patient to take which of the following precautions prior to the test?

remove jewelry Rationale CT or CAT= Computerized (axial) Tomography. A CT is a non-invasive test that provides detailed, cross-section views of all tissue types. Metal objects may interfere with image results, so the patient should remove all jewelry before the test. The patient does not need to wear a mask, avoid caffeine, or remove contact lenses, because none of these would interfere with the test.

Which of the following arteries circulates blood to the kidneys?

renal Rationale A significant portion of blood flow to the kidneys is supplied by the renal arteries. The femoral arteries are in charge of supplying blood to the legs, the carotid arteries supply the brain with blood, and the brachial arteries supply blood to both arms.

When a patient is diagnosed with Benign Prostatic Hyperplasia (BPH), which of the following body systems is affected?

reproductive Rationale Benign prostatic hyperplasia--also called BPH--is a condition in men in which the prostate gland is enlarged and not cancerous. Benign prostatic hyperplasia is also called benign prostatic hypertrophy or benign prostatic obstruction. A common male reproductive system condition that occurs as men age. As the prostate enlarges, the gland presses against and pinches the urethra. The bladder wall becomes thicker. Eventually, the bladder may weaken and lose the ability to empty completely, leaving some urine in the bladder. The respiratory, skeletal and nervous system are not involved with this diagnosis.

Vital signs documented on an older adult patient in the outpatient clinic are recorded as: Temperature: 98.6F (37.0C), Pulse: 68, Respirations: 30, B/P 128/84 mmHg. Which of the following measurements should the medical assistant reassess?

respirations Rationale The normal respiratory range for an adult is 14-20, therefore a RR of 30 would indicate tachypnea (increased RR). The temperature, pulse, and BP are all within normal limits, but the RR should be reassessed.

The medical assistant caring for a 6-month old infant in the clinic would measure vital signs in which order?

respirations, pulse, blood pressure, temperature Rationale For infants, toddlers, uncooperative preschoolers, or children with known delays, it is best to begin with the least invasive procedure and leave uncomfortable procedures for last (e.g. blood pressure, temperature, diaper change). The medical assistant would first count respirations, then check the pulse and take blood pressure. The temperature check would be last since it is the most invasive.

The pharynx is part of which of the following body systems?

respiratory Rationale The pharynx is part of the respiratory system. The upper respiratory tract includes the nose, pharynx (throat), and larynx (contains vocal cords). The lower respiratory tract includes the trachea (windpipe), bronchioles (large and small airway branches), and lungs. The skeletal system includes the bones, the nervous system includes the brain, spinal cord, and nerves, and the sensory system is also part of the nervous system.

The patient in the cardiac clinic is diagnosed with cor pulmonale. Which of the following conditions best describes this disorder?

right-sided heart failure Rationale Cor pulmonale, also known as right-sided heart failure, occurs after prolonged hypertension in the right ventricle of the heart. Increased blood pressure in the right ventricle makes it difficult for blood to be pumped to the lungs, therefore blood builds up in the right side of the heart. Left-sided heart failure is the most common type of congestive heart failure and is caused when the left side of the heart cannot pump blood out effectively. Hypertension and hypotension refer to high and low blood pressure, respectively.

The humerus is proximal to which of the following?

scapula Rationale Basic anatomy, rationale not applicable.

Which of the following physical characteristics may be present in a patient with Down Syndrome?

single palmar crease Rationale Also known as Trisomy 21, a congenital defect that causes abnormalities with the body's brain and physical development causing a single palmar crease. Cleft lip and cleft palate are birth defects that occur when a baby's lip or mouth do not form properly during pregnancy. Clubbed fingers is a symptom of disease, often of the heart or lungs which cause chronically low blood levels of oxygen. Turner syndrome is a chromosomal condition that alters development in females, one symptom is the wide chest with broadly spaced nipples.

Which of the following conditions describes inflammation and/or infection of the sinus cavity?

sinusitis Rationale Sinusitis, inflammation of the sinus cavity, affects the mucous membranes that line the nose and sinuses. It may occur with viral, bacterial, or fungal infections or be due to allergies. Sinus arrhythmia (heart rate changes during breathing) and sinus tachycardia (increased heart rate) refer to changes in heart rate. And conjunctivitis, also known as pink eye, is inflammation of the conjunctiva in the eye.

Reminding patients to refrain from talking and moving, and keeping them warm during the ECG procedure are important to help eliminate

somatic tremor artifact. Rationale Movement (shivering, blatant movements, talking) during an ECG procedure can cause somatic tremor artifact. A wandering baseline would be seen if a lead fell off, whereas electrical interference could cause the 60-cycle interference.

Which of the following types of fractures is caused by the break winding around the bone?

spiral Rationale A spiral fracture is caused by a twisting force, thus leading to a break that winds/twists/spirals around the bone. Buckle fractures (a.k.a. torus fractures and greenstick fractures), common in children, are incomplete fractures that essentially bend but don't fully break. Compound fractures are injuries that actually break through the skin.

Which of the following organs is most likely to be enlarged and tender in a patient with acute mononucleosis?

spleen Rationale The spleen is in charge of helping to fight off infection within the body, and when there is an infection it will become enlarged in order to try to fight off whatever is harming the body. Therefore, in a patient with acute mononucleosis, the spleen will be enlarged and will also be tender. This enlargement indicates that the body has activated the immune response to help fight the infection. Mononucleosis will not cause the pancreas, liver, or gallbladder to be enlarged.

The medical term meaning immediately is

stat. Rationale The medical term meaning immediately is stat. Ad lib means to do as desired, prn is a medical term meaning as needed, and asap means as soon as possible.

When preparing the sterile tray for a laceration repair, the medical assistant should add which of the following items? (Select the two (2) correct answers.)

sterile 4x4 gauze pads, sutures Rationale Sterile gauze pads and sutures are sterile items. The vial container is non-sterile and will be used prior to the procedure. Bandage scissors and antibiotic ointment are used following the procedure.

Which of the following describes a wound affecting only the epidermis?

superficial Rationale The prefix super- means above, or superior. The epidermis is the outer layer of skin, thus a superficial wound (abrasion) affects only the epidermis. Complex, penetrating, and through-and-through lacerations would all be deeper than the epidermis.

The suffix -ectomy means

surgical removal. Rationale The suffix -ectomy means to cut, excise, or surgically remove. Ex. A tonsillectomy is the surgical removal of the tonsils. The suffix for the surgical repair of something is -plasty, -stenosis is the suffix for the terms stricture and narrowing.

A patient is diagnosed with lymphedema has which of the following conditions?

swelling of the lymphatic tissue Rationale The definition of Lymphedema is Lymph=lymphatic, edema= swelling. Leukopenia is a lowered white blood cell count and Lymphoma is a group of blood cell tumors that develop from lymphocytes.

The prefix meaning abnormally fast is

tachy-. Rationale The prefix that means abnormally fast is tachy. Exp. A respiratory rate above the normal range is described as tachypnea. Macro- is a prefix meaning large, multi- means many, and brady- means abnormally slow.

An abnormally rapid heart rate is described as

tachycardia. Rationale The prefix tachy- means fast, and the suffix -cardia refers to the heart, therefore tachycardia means an abnormally rapid heart rate. Hypertension is a term referring to high blood pressure, tachypnea means abnormally rapid respiratory rate, and hyperplasia is a term referring to increased cell production.

The medical assistant finds an unconscious patient in the office building hallway and determines CPR needs to be initiated. Which of the following allows the medical assistant to begin CPR?

the Good Samaritan Law Rationale If an emergency treatment is given in good faith, and care is administered without gross negligence, then this is considered under the Good Samaritan Law. The Patient is passed out and cannot imply consent or actively consent to anything. Implied Consent is a legally enforceable agreement that arises from assumed intentions, (i.e. putting an arm out for a venipuncture implies consent for that procedure).

The patient complains of ringing in the ear. What is the medical term used to describe this symptom?

tinnitus Rationale The medical term for ringing in the ear is tinnitus. It is a common condition that is often described as a noise, swishing, or ringing in the ear. Vertigo is the medical term for dizziness, ataxia is the loss or lack of coordination, and rhinitis is the inflammation of the mucosa in the nose.

What is the importance of encouraging a patient diagnosed with bronchitis to maintain adequate fluid intake?

thin secretions Rationale Bronchitis is an inflammation of the bronchial tubes, which are in charge of taking air to the lungs. When they get inflamed, thick secretions build up in the bronchial tubes. Patients with bronchitis need to maintain adequate fluid intake in order to thin the secretions they have. Adequate fluid intake will also replace insensible losses and will help to prevent dehydration. By increasing fluid intake and thinning the secretions, the airways will be clearer and there will be less fatigue. Antibiotics are typically prescribed to decrease the inflammation.

The physician asks the medical assistant to calculate a patient's BMI after taking the weight and height. What is the importance of this measurement?

to estimate the patient's body fat and obesity-related health risks Rationale Body Mass Index (BMI) is a relationship of weight to height: BMI = weight [kg]/height [m2] (weight is in kilograms; height is in meters, which is squared in the calculation). 75kg/1.7m2 = 75/2.89 = 26 BMI A high BMI can be an indicator that a patient is overweight or obese. There are increased chronic health risks associated with overweight and obese individuals (diabetes, high blood pressure, etc.). The BMI is one measurement that helps determine an overall risk factor for obesity-related health issues. It is not a screen for diabetes (a glucose tolerance test, fasting glucose, triglycerides, and Hgb A1C would help determine if a patient is diabetic) or related complications. The BMI is not used to detect hydration levels or skin fold thickness.

The medical term gravida describes which of the following?

total number of pregnancies Rationale The term gravida refers to the total number of pregnancies a woman has. Ex. A woman's first pregnancy is referred to as gravida 1(primigravida), second, gravida 2, and so on. The onset of menses is call menarche, the last menstrual period is called menopause, and the total number of live births is called para.

Referring to the Illustration, identify number 87. (diamond shape muscle back of neck)

trapezius Rationale Basic anatomy, rationale not applicable.

Which of the following terms describes the ear drum?

tympanic membrane Rationale The prefix tympan/o means ear drum, and is typically abbreviated as TM. The tympanic membrane normally appears pearly grey in color when viewed with an otoscope. A myringotomy tube is another name for an ear tube, the auditory canal is the tube that connects the outer ear to the tympanic membrane (ear drum), and the Eustachian tube connects the nasopharynx to the middle of the ear.

The pediatric patient is brought to the office with an obvious deformity and pain in the right lower arm after a fall from a bunk bed. This injury affects the

ulna. Rationale The ulna is located in the forearm, parallel to the radius. The femur is the located in the thigh, the humerus is located in the upper arm and the clavicle runs parallel, across the front of the shoulders (also known as the "collarbone").

The ulna is part of the

upper extremity. Rationale Basic anatomy, rationale not applicable.

The radius is part of the

upper extremity. Rationale The radius is one of two long bones of the forearm (upper extremity), extending from the elbow to the thumb side of the wrist. The lower extremities are the legs, and the anterior fossa and posterior fossa are in the skull.

Which of the following structures is part of the genitourinary system?

urethra Rationale The GU (renal or urinary) system also includes bilateral kidneys, ureters, and bladder. The cervix, uterus, and fallopian tubes are part of the reproductive system.

Which of the following tests is most likely to be performed in the patient diagnosed with pyelonephritis?

urine c&s Rationale Pyelonephritis (kidney infection) is an infection of the kidneys. Submission of a urine sample for bacterial culture and sensitivity testing would provide the physician with information to help guide treatment. Hemoccult testing is utilized to help detect blood in a stool specimen. Rh factor is a blood test to indicate presence or absence of the Rhesus factor on the surface of red blood cells. There is no indication for hemolytic anemia in this patient, therefore a DAT (direct antiglobulin test) is not a worthy choice in this scenario.

The prefix in the term phlebitis refers to inflammation of which of the following?

veins Rationale The prefix phlebo(o)- means pertaining to a vein, which would mean the term phlebitis is the inflammation of the vein. The inflammation of an artery is called arteritis, the inflammation of the appendix is referred to as appendicitis, and pancreatitis is the inflammation of the pancreas.

Which of the following vitamins is most important to bone health?

vitamin D Rationale Vitamin D promotes calcium absorption, and enables adequate bone growth and development. Also prevents Rickets in children. Vitamin A is a fat-soluble vitamin that acts as an antioxidant and may help reduce the risk of cancer. Vitamin C, also known as L-ascorbic acid, is a water-soluble vitamin that is naturally present in some foods and is required for the biosynthesis of collagen and certain neurotransmitters. Vitamin K is a fat-soluble vitamin that plays important role in blood clotting and building strong bones.

Which of the following measurements should be obtained on an infant in the office for a well-child visit? (Select the three (3) correct answers.)

weight, length, head circumference Rationale Weight, length and head circumference are documented on a standardized growth chart until 2 years of age.

Abnormally dry skin is described as

xerosis. Rationale Abnormally dry skin is described as xerosis. Vitiligo is a skin condition where there is a loss of pigment from certain areas, excoriation is a scratch or injury to the skin, and ecchymosis refers to the brusing of the skin.


संबंधित स्टडी सेट्स

WEEK 9 ASSIGNMENT MCB3020 BACUSMO

View Set

World Civilizations chapters 10-11

View Set

nursing 124; gallbladder disease (mod 10)set #2

View Set

Construction Project Management Exam 2 (quizzes)

View Set

chapter 26 Electrolytes, water, and acid base balance

View Set

ga life, accident, disability and health insurance license final review

View Set

Business Law Ch. 19 all sections

View Set

organic and biological compounds

View Set

2.1 - Economic Systems - Instruction

View Set